You are on page 1of 49

CHAPTER

Redox Reactions and


9 Electrochemistry

Chapter Highlights
Oxidation, Reduction, Redox reactions, Oxidation number, Balancing of oxidation reduction equations,
Electrolysis, Faraday’s law, Electrical conductance, Conductivities, Kohlraush law, Voltaic cell, Electrode poten-
tial, emf, Nernst equation, Some commercial cells, Corrosion and various levels of multiple-choice questions.

REDOX REACTIONS

BASIC CONCEPTS Reduction


Oxidation Reduction involves any one of the following processes.
• Remtoval of oxygen
Oxidation involves any one of the following processes. PbO + C Pb + CO
• Combination of oxygen
• Addition of hydrogen
2Mg + O2 2MgO
H2 + Cl2 2HCl
• Removal of hydrogen
• Removal of electronegative element or decrease
4HCl + MnO2 MnCl2 + Cl2 + 2H2O
in its ratio.
• Addition of electronegative element or increase 2FeCl3 2FeCl2 + Cl2
in its ratio
• Addition of electropositive element or increase in its
2SnCl2 + Cl2 2SnCl4
ratio.
• Removal of electropositive element or decrease 2HgCl2 Hg2Cl2 + Cl2
in its ratio
• Gain of electrons by an atom or an ion.
Hg2Cl2 HgCl2 + Hg
Cu2+ + e– Cu+
• Loss of electrons by an atom or an ion or a molecule. • Decrease in oxidation number of metal atom.
Na – le– Na+
Fe2+ – le– Fe3+ Redox Reaction
• Increase in oxidation number or positive charge on • Here oxidation and reduction take place simultane­
the metal atom. ously, that is, one species gets oxidized while the
• Rusting of iron and combustion are other instances other one gets reduced.
of oxidation. +3 +2 +2 +4
2FeCl3 + SnCl2 2FeCl2 + SnCl4
9.2 Chapter 9

Here SnCl2 is oxidized into SnCl4 while FeCl3 is Reducing Agents or Reductants
reduced into FeCl2. Reducing agents are species which reduces another but
n+ n+
A1 + A2 A1 + A2 gets oxidized itself.
n+ n+
Here A1 is oxidized into A1 while A2 is reduced Some common reductants
into A2.
• Metals of group IA (Li, Na etc.) and group IIA
• All redox reactions are exothermic in nature. (Mg, Ca etc.)
Redox reactions can be classified into following three • Non metals like C, S, P, H
types, viz., intermolecular, disproportion and intramolec­
• HX like HI, HBr, HCl
ular reactions.
• Metal hydrides (MH) like LiH, CaH2
Intermolecular redox reactions • Molecules or compounds in which the element has
One species gets oxidized while another gets reduced. lowest oxidation state, (e– needed to achieve octet state).
For example, +1 –2 +1
For example, Hg2Cl2, H2S, Cu2O, SnCl2, FeCl2
+2 +2

2HgCl2 + SnCl2 Hg2Cl2 + SnCl4 • Some organic acids like HCOOH, (COOH)2
Here SnCl2 is oxidized into SnCl4 while HgCl2 is
reduced into Hg2Cl2. Oxidation Number
Oxidation number is the residual charge (±) present on an
Auto redox or disproportion reaction
atom in a given molecule or ion when the shared electrons
A single species undergoes oxidation as well as reduction are counted towards more electronegative atoms.
simultaneously.
Examples: Species Which can Act Both
2Cu+ Cu2+ + Cu0 as Oxidants and Reductants
0 +1 –1
X2 + 2OH

XO + X
– –
Here the value of the oxidation number of the element
+1 –3 +5 lies between a maximum and a minimum limit, which
2H3 P O2 PH3 + H3PO4 means, it can be oxidized as well as reduced. Hence,
Cannizzaro Reaction it can act both like an oxidant as well as a reductant.

R–CHO OH R–CH2OH + R–COO– +3 +2 +4 –1
For example, H NO2, CO, SO2, H2O2
Intramolecular redox reactions
Here, in a compound, one element undergoes oxidation
Rules to find oxidation number
while the other undergoes reduction. For example,
+5 –1 0
• Oxidation number of all neutral atoms, molecules,
2K ClO3 2KCl + 3O2
compounds, complexes, polymeric form is zero, for
Here oxygen undergoes oxidation while chlorine under­ example, Na, O2, P4, O3, S8, KMnO4
goes reduction.
−3 −5
• Oxidation number of any ion = charge on ion
NH 4 NO 3 → N 2+1O + 2H 2 Ot Examples:

Oxidizing Agents or Oxidants 1. CN–1 = –1


Oxidants are species which oxidizes others but itself gets 2. NH+4 = +1
reduced. For example, 3. SO–2
4
= –2
PbS + 4O3 PbSO4 + 4O2 4. PO4 = –3
3–
Oxidizing
agents
• H has +1 oxidation state when it forms compounds
Some common oxidants with non­metals but with metals in metal hydrides
• Molecules of most electronegative elements, for (MH), it has an oxidation number of –1.
ex ample, O3, O2, F2, Cl2 etc. Examples:
• Elements in their highest oxidation states in com­ l. H2O = +1
pounds. For example, 2. LiH = –1
+7 +6 +6 +5 +6 +4 +3
KMnO4, K2Cr2O7, H2SO4, HNO3, SO3, CO2, FeCl3 3. CaH2 = +l
• Some oxides like MgO, CaO, CrO3, H2O2.
Redox Reactions and Electrochemistry 9.3

• Oxidation number of alkali metals (group IA) is always Group VIA elements show –2 to +6 oxidation states
+1 and oxidation number of alkaline earth met als Group VIIA elements show –1 to +7 oxidation states
(group IIA) is always +2 in their respective com­
pounds, for example, NaCl, Na2O, MgO, Ca(OH)2 FA C T S TO R E M E M B E R
+1 +1 +2 –2

Oxidation number of oxygen 1. Maximum oxidation number of +8 is shown by Os,


• Oxidation number of oxygen is –2 in its compounds Ru in OsO4, RuO4.
with metals as well as non­metals, for example, 2. Oxidation number can also be a fraction
CO, MgO Examples:
• In peroxides (M2O2) such as Na2O2 and (MO2), such 1
N3H = – (for N-atom)
as BaO2, the oxidation number of oxygen is –1. 3
• In superoxides (MO2) such as KO2, CsO2, RbO2 oxi­ 8
dation number of oxygen is –½. C3H8 = – (for C-atom)
3
• In case of O2F2 and OF2 oxidation number of oxygen
is +1 and +2 respectively. 8
Fe3O4 =+ (for Fe-atom)
3
Oxidation number halogens
• Fluorine always has –1 oxidation state in its com­
pound such as HF, OF2. Some Special Examples of Oxidation State
• Chlorine has +1 to +7 oxidation state with fluo­ 1. In CaOCl2, the oxidation number of Cl atom is
rine or oxygen but –1 commonly with metals and +1 and –1.
non­metals. 2. In H2SO5, H2S2O8, the oxidation number of S atom
For example, HCl, NaCl, MgCl2 (–1 state) is + 6.
ClO ..................... Cl2O7 O
+2 +7
H–O–S–O–O–H
• Bromine can have –1 to +6 oxidation states like
O
chlorine, for example, O O
HBr, BrF3, BrO3
–1 +3 +6 H–O–S–O–O–S–O–H
• Iodine can have –1 to +7 oxidation states, like chlo­ O Peroxy O
linkage
rine, for example, HI, IF3, I O–4
–1 +3 +7 3. In CrO5 the oxidation number of Cr atom is +6.
Oxidation number of metals in metal carbonyls O
O O
In metal carbonyls [M(CO)n], such as Ni(CO)4, [Fe(CO)6], Cr
and metal amalgams, NaHg, ZnHg oxidation number of O O
metal atom is always zero.
4 In KO3 (Potassium ozonide) the oxidation number
of oxygen is –1/3.
NOTE 5. In carbohydrates with empirical formula CH2O,
the oxidation number of C atom is zero.
If any compound has [NH3, CO, NO, H2O etc.] In then the 6. In all the allotropes of an element, the oxidation
value of oxidation number for the these is put as zero, for number of the element is always zero.
example in Zn(NH3)2SO4,
7. Oxidation number of C atom, in both HCN and
X+0–2=0 HNC, is +2.
X=+2 8. In Na2S2O3, the oxidation numbers of S atom are
–2 and +6.
±
Range of oxidation number  ± 2 
Maximum oxidation number of any element is equal to its 1D2±6±21D
group number while minimum is equal to group number –8.  ±
6
Group IVA elements show –4 to +4 oxidation states ±
Group VA elements show –3 to +5 oxidation states
9.4 Chapter 9

For example, for H3PO4


9. In Na2S4O6, the oxidation number of two S atom
is zero, while the other two S atoms have +5 as Ca(OH)2 + H3PO4 CaHPO4, + 2H2,O
the oxidation number. Here, E = M/2
10. NOCl NH4 NO3
x + (–2) + (–1) = 0 NH4+ NO3– BALANCING OF REDOX REACTIONS
x = +3 x + 4 = + 1 x – 6 = –1
x = –3 x = +5 By Ion-Electron Method
• Ion­electron method is also called half-reaction
Application of oxidation number
method.
• Acidic nature of oxides or oxy acid s of non­metals, • Ion­electron method of balancing a redox reaction
is directly proportional to oxidation number of the involves following steps one by one:
central atom.
Examples: 1. First write the half equations.


+2
CrO
+3
Cr2O3
+6
CrO3 2. Try to balance each atom from the beginning.

Increasing order of acidic nature 3. I n case of acidic medium add H+ to the side with
more oxygen atoms or less positive charge and
+4 +6
SO2 < SO3 add H2O to opposite side of H+.
+1 +4 +5 +7 For example Mn O 4–+8H+ Mn2+ + 4H2O
HOX < HXO2 < HXO3 < HXO4
4. I n case of basic medium or when more negative
Increasing order of acidic nature charge is present on right hand side of equation,
–1/3 +1 +3 +5 add OH– to the side with lesser number of oxygen
N3H < H2N2O2 < HNO2 < HNO3
atoms or the less negatively charged side.
• It is also used to find equivalent weight. For example X2 + 12OH– 2XO3– + 6H2O
Molecular weight
E= ____________________
Change in oxidation number 5. T
o balance charges on both the sides, add or subtract
suitable number of electrons as follows
1. For example, for the following reaction 5e– + MnO–4 + 8H+ Mn2+ + 4H2O

2–
Cr2O7 +14H+ + 6e– 2Cr3+ + 6H2O As left hand side has +7 units charge while right
hand side has just +2 units of charge,
E=M/6.
X2 + 12OH– 2XO3– + 6H2O + 10 e–
2. Taking KMnO4 as an example, the equivalent weight
of manganese is calculated as follows: 6. Now to cancel out the electrons, multiply either
(a) In acidic medium, E = M/5 one equation or both the equations with suitable
KMnO4 + 3H2SO4 K2SO4 + 2MnSO4 numbers.
+ 3H2O + 5 [O]
7. F
inally, add the balance half equations and see
As Oxidation number changes by 5 units. that all atoms and charges are balanced on both
(b) In neutral medium, E = M/1 the sides.
2KMnO4 + 2KOH 2K2MnO4 + H2O + [O]
Example 1
As Oxidation number changes by 1 unit.
(c) In neutral medium, E = M/3 X2+OH– X– + XO3
2KMnO4 + H2O 2KOH + 2MnO2 + 3[O] Solution
As Oxidation number changes by 3 units. First, write half equations and balance these, one by
3. For acidic salts one.
Molecular weight 2e– +X2 2X– (i)
E = ______________________
Number of replaceable H atoms Here RHS has 2 unit negative charge, so on LHS
2e– are added to balance the charge on both sides.
Redox Reactions and Electrochemistry 9.5

X2+12OH– 2XO–3+6H2O+10e– (ii) 3. To equalize the increase and decrease in oxida­



Here OH is taken opposite to XO 3 and 10 e are
– – tion nu mber, multiply both equations with suitable
added to balance negative charge on both sides. numbers.
Now to cancel electrons in both these equations 4. Now balance the complete equation after adding both
multiply equation (i) by 5 and adding both equations half equations.
we get
5. Addition of H+, OH– and H2O is the same as in
10e– +5X2 10X– ion­electron method.
X2 + 12 OH 2XO–3 + 6H2O + 10e–
6X2+12 OH − 2XO–3 + 10X– + 6H2O FA C T S TO R E M E M B E R
or
n-factor It is the number of electrons lost or gained in
3X2 + 6 OH– XO–3 + 5X– + 3H2O half-reactions. It is always equal to the difference of oxida-
This is a balanced chemical equation, as both sides have tion number of the element which undergoes a change in
the same number of atoms of each element, as well as oxidation number. Examples:
charges. MnO4– Mn2+
Example 2 Here n-factor = 7 – 2 = 5
MnO–4 +C2O2– Mn2+ + CO2 Cr2O72– 2Cr3+
4
Solution Here n-factor =12 – 6 = 6
First write the half equation as follows:
MnO–4 Mn2+ Example 1

As positive charge is greater on RHS, so add H+ to LHS Cr2O72– + Fe2+ Fe3+ + Cr3+ + H2O
and H2O to RHS as follows:
Solution

MnO +8H
4
+ 2+
Mn +4H2O Cr2O7–2 + Fe2+ Fe3+ + Cr3+ + H2O
Now to balance charge on both sides, add 5 electrons he half­reactions and n­factor (change in oxidation
T
on LHS to get a balanced half equation number) can be shown as
Cr2O72– 2Cr3+ (i)
5e– + MnO–4 + 8H+ Mn2+ + 4H2O (i)
n­factor = 12 – 6 = 6
Now write another half equation; Fe2+ Fe3+ (ii)
C2O2– 2CO2 + 2e– (ii) n­factor = 3 – 2 = 1
4
After the cross multiplication of n­factors (to make
Here atoms are balanced by writing 2 before CO2, change of oxidation number equal in both half equa­
while charge is balanced by adding 2 electrons on RHS. tions), we get
Now to cancel the electron in both the equations, multi­
Cr2O72– + 6 Fe2+ 2Cr3+ +6 Fe3+
ply equation (i) by 2 and equation (ii) by 5. On adding both
equations, we get inally to balance number of O, H atoms we will add
F
14 H+ to LHS and 7 H2O molecules to RHS.
2MnO–4 +5C2O2–
4
+16H+ 2Mn2+ + 10CO2+8H2O
Cr2O72– + 6 Fe2+ + 14 H+ 2Cr3+ +6 Fe3++ 7H2O
This is a balanced equation in terms of atoms as well as
This is the balanced equation.
charges.
Example 2
By Oxidation Number Change Method Cl2 + I O 3– + OH– I O4 – + Cl– + H2O
Solution
1. First find oxidation number of those atoms, which 0 +5–2 –2 +1 +7 –2 –1 +1 –2
show a change in their oxidation numbers. Cl2 + I O 3– + OH– I O4 – + Cl– + H2O
2. Make two half equations, according to increase or The half­reactions and n­factor (change in oxidation num­
decrease in oxidation numbers. ber) can be shown as
9.6 Chapter 9

0 –1 Cl2 + I O 3+ I O3 + + 2Cl–
Cl2 2Cl– (i) Finally, to balance number of O, H atoms we will add 2
n­factor = 2 – O = 2 OH– to LHS and one H2O molecule to RHS
+5 +7
Cl2 + I O –3 +2OH– I O–4 + 2Cl– + H2O
I O 3– I O 4– (iii)
n­factor = 7 – 5 = 2 This is the balanced equation.
There is no need of cross multiplication of n­factors (as
change in oxidation numbers are equal in both half equations)
so,

ELECTROCHEMIESTRY
Electrochemistry is the branch of science that deals with • If active electrode is used, metal gets deposited at
the transformation of chemical energy into electrical cathode and is dissolved at anode.
energy and vice versa. It deals with the relationship • When fused salts are electrolyzed by using inert
between electrical and chemical energy produced in a redox electrodes, they ionize into their constituent elements.
reaction.
For example, NaCl Na + ½Cl2
NiCl2 Ni + Cl2
ELECTROLYSIS
• When fused salts are electrolyzed by using active
In electrolysis, direct current is used to cause a non­sponta­ electrodes, anode gets dissolved due to formation of
neous chemical reaction, due to which chemical decompo­ ions, while cathode increases in size due to deposi­
sition of electrolytes takes place. tion of metal on it.
It is the flow of electrons across the boundary, followed
by a chemical reaction, that is, redox reaction. • During electrolysis of aqueous solution of AgNO3,
the concentration of AgNO3 is found to be same,
• It takes place at the electrodes.
before and after electrolysis.
• It occurs in electrolytic cell.
• During the electrolysis of aqueous solution of salts
• The electrode which is connected to the positive
pole of the battery is known as anode or positive having NO–3, and S2−
4 (oxo anions), only water mole­

electrode. cule undergoes oxidation to liberate oxygen at anode.


• At anode, de­electronation of anions or oxidation • During electrolysis of aqueous solution of alkali or
takes place. alkaline earth metals, only water molecules undergo
A– A + e– reduction to liberate hydrogen at cathode.
• The electrode which is connected to the negative For example,
pole of the battery is known as cathode or negative Electrolysis of molten or fused NaCl
electrode. Anode 2Cl– Cl2 + 2e–
• At cathode, electronation of cation or reduction takes Cathode 2Na+ + 2e– Na (s)
place.
Net reactions
B+ + e– B
• It is interesting to note that the current enters the 2Na+ + 2Cl– Cl2 (g) +2Na (s)
electrolyte through cathode and leaves through
anode.
Table 9.1 Features of Cathode and Anode
• The electrolyte as a whole remains neutral during the Feature Cathode Anode
process of electrolysis, as equal number of charges Sign Negative as Positive as attached
are neutralized at the electrodes. attached to to positive end of
• Electrodes are of two types: (i) inert and (ii) active negative end of external battery
external battery
• If inert electrodes are used, the products formed at
cathode and anode depend upon nature of electrolytes.
Continued
Redox Reactions and Electrochemistry 9.7

Table 9.1 Features of Cathode and Anode Electrolysis of H2SO4


Feature Cathode Anode For pure H2SO4, electrolysis is not possible, due to absence
of ions.
Direction of Into the cell Out of the cell
H2SO4 2H+ + SO4–2
movement of
electrons H2O H+ + OH–
Direction of Cations Anions
movement of
ions H2 at –ve electrode O2 at +ve electrode
Half-reaction Reduction Oxidation
The solution becomes colourless and pH decreases, as
H2SO4 is formed.
Electrolysis of Aqueous NaCl Solution If further electrolysis is carried out, O2 is released at anode
and H2 is released at cathode.
NaCl Na+ +Cl–
2Cl– Cl2 + 2e– at positive electrode Electrolysis with Active Electrodes
H2O H+ + OH– • When an aqueous solution is oxidized by using metal
2H+ + 2e– H2 at negative electrode electrodes, the metal electrode will be oxidized as its
NaOH is in solution, so pH increases EO.P. is more +ve than that of water

At anode, CI– donates e–, as EO.P. of Cl– >E O.P. of OH– Ni(s) Ni2+ (aq) + 2e– (EO.P. = +0.28 V)
At cathode, H+ accepts e–, as ER. P. of H+ > E R. P. of Na+ 2H2O O2(g) + 4H+ (aq) + 4e– (E = –1.23 V)
In this case O2 is not formed at anode, as voltage is very • If Ni deposits at cathode it is called electroplating.
high for it.

Table 9.2 Electrolysis of Some Electrolytes


S.No. Electrolytes Electrode Cathode Products At Electrode reactions At cathode
Anode At anode cathode At anode
1 NaCl (fused) Pt Pt Cl2 Na 2Cl– Cl2 + 2e– Na+ + e– Na
– – 2+ –
2 MgCl2 (fused) Graphite Steel Cl2 Mg 2Cl Cl2 + 2e Mg + 2e Mg
3 Al2O3 (fused) Graphite Graphite F2 Al 2F – F2 + 2e– Al3+ + 3e– Al
+ cryolite
4 NiCl2 (fused) Pt Pt Cl2 Ni 2Cl– Cl2+2e– Ni2+ + 2e– Ni
5 NaCl (aq) Pt Pt O2,Cl2 H2 2H2O O2 + 4H++4e– 2H2O + 2e– H2
2Cl– Cl2 + 2e– +2OH–
Pt O2 Ag 2H2O O2 + 4H++4e– Ag+ + e– Ag
6 AgNO3 (aq) Pt Pt O2, Ag+ Ag 2H2O O2 + 4H++4e– Ag+ + e– Ag
Ag Ag+ + e– Ag

7 CuSO4 (aq) Pt Pt O2 Cu 2H2O O2 + 4H+ + 4e– Cu2+ + 2e– Cu


8 50% H2SO4 Pt Pt H2S2O8 H2 2HSO4– H2S2O8 +2e– 2H+ +2e– H2
(aq)
9 K2SO4(aq) Pt Pt O2 H2 2H2O O2 + 4H+ + 4e– 2H2O + 2e– H
– 2
+2OH
10 CuCl2 Pt Pt Cl2 2Cl– Cl2 + 2e– Cu2+ + 2e– Cu
(molten) Cu Pt Cu2+ Cu Cu2+ + 2e– Cu2+ + 2e– Cu
9.8 Chapter 9

Second Law of Electrolysis


FA C T S TO R E M E M B E R
According to second law of electrolysis, “When the same
 The apparatus used for the determination of a substance quantity of electricity is passed through different electro­
released during electrolysis by measuring the quantity of lytes, the weight of the products obtained at the electrodes
electrical charge is called coulometer. is directly proportional to their chemical equivalents or
 The amount of ion discharged during electrolysis is inde- equivalent weights.”
pendent of resistance of solution and temperature of
bath. W
As = Constant
 The art of electroplating was developed by Faraday. E

E1 M1 Z1 .I.t Z1
FARADAY’S LAWS So = = =
E 2 M 2 Z 2 .I.t Z 2
First Law of Electrolysis
E1 = Equivalent weight mass
According to first law of electrolysis, “The amount of E2 = Equivalent weight mass
substance deposited or quantity of chemical reaction at an
M = Mass deposited
electrode, is directly proportional to the quantity of elec­
tricity passed through the cell.” Application of Faraday’s Laws
w or m α Q
• In electroplating of metals.
or w α It
• In the extraction of several metals in their pure form.
w = Z.I.t • In the separation of metals from non­metals.
M
Z = ___
nF • In the preparation of compounds.
Z = Electrochemical equivalent
TERMS AND CONCEPTS RELATED TO
M = Molar mass
ELECTROCHEMISTRY
F = 96500 (Faraday constant)
n = Number of electrons transfered Current Efficiency
• Electrochemical equivalent (Z) is the amount of the It is the ratio of the mass of the products actually liberated
substance deposited or liberated by one ampere cur rent at the electrode to the theoretical mass that can be obtained.
passing through it for one second (that is, one coulomb) Desired extent
C.E. = _____________________
Theoretical extent of reaction ×100
• One g equivalent of any substance is liberated by one
faraday. Electrical Conductance
1 faraday = 1 coulomb of electricity or one ampere
• In electrolysis, conductance is due to movement of
per second.
ions through electrodes. It involves transfer of matter
Equivalent weight = Z × 96500 and ions. It occurs in a small magnitude.
W _____
__ Q • It depends upon
E = 96500 solute­solute interaction
E.Q solute­solvent interaction
w = _____
96500
viscosity of solvent
E.I.t
w = _____
96500 conductance α temperature
(as kinetic energy α temperature)
FA C T S TO R E M E M B E R Ohm’s law According to this law,
One faraday is the quantity of charge carried by one mole E
__
of electrons. I = constant (R)
or
EαZ E
I = __
R
E = FZ
1F = 1.6023 × 10–19 × 6.023 × 1023 E = Potential, R = Resistance
= 96500 coulombs (1 gm - equivalent) Unit is ohms (Ω)
Redox Reactions and Electrochemistry 9.9

Resistance (R) w NV
As __ ____
E = 1000
It is the obstruction offered to the passage of electric
k × 1000
current. It is directly proportional to the length (l) and So, λ = _______
N
inversely proportional to the area of cross section (a) of Equivalent conductivity increases with the increase
the conductor. of dilution.
R ∝ l/a or R = ρ _a
l
Molar conductivity
Specific resistance (ρ) The conductivity of all the ions produced, when one mole
The resistance offered by one cm3 of the conductor is of an electrolyte is dissolved in V mL of solution is known
known as specific resistance. as molar conductivity.
If l = 1 cm and a = 1 cm2 1000 × k × v
Then R = ρ λm or Λm = __________
M (molarity)
a
As ρ = R __l ohm cm
Effect of dilution on molar conductivity
Conductance
Molar conductivity increases with dilution, as degree
(G or C) It is the ease of flow of electric current through the of dissocia tion of electrolyte increases with dilution. At
conductor and is reciprocal of resistance. infinite dilu tion, Λ is maximum.
1
C or G = __
R
Λν
Units are Ω– (mho) α=
1S = 1Ω– Λ∝
siemens Debye-Huckel-Onsager equation
Specific conductance It shows a relation between molar conductiity at a par­
(k, kappa) It is the reciprocal specific resistance. ticular oncen­tration and molar concentration at infinite
_l dilution as fol lows:
1
k = __ 1
__
ρ = R × a
_l Λ m = Λ ∝m − b C or µ = µ ∝ − b C
a = cell constant
Units Ω– cm–1 where
b = Constant
If l and a are unity, then conductance of one cm3 of
C = Concentration
solution is called specific conductance or conductivity
µ∞ = Molar conductance at infinite dilution
Effect of dilution on specific conductivity The value of b for a given solvent and temperature
The number of current carrying particles or ions per mL depends upon the nature of electrolyte that is, the charges
decrease on dilution, and specific conductivity, being the on cation and anion produced after dissociation. All elec­
conductance of one cubic centimeter of solution, decreases trolytes of a particular type have same value of b.
with dilution.
Specific conductance Factors affecting molar conductivity
Cell constant = ________________
Measured conductance 1. Nature of electrolyte: Strong electrolytes like HCl,
Conductance is measured by using wheat stone H2SO4, NaOH, KOH, KNO3, KCl, NH4NO3 are com­
pletely ionized in aqueous solution, so they have a high
bridge method (for R) and IN or 0.01 N KCl solution
value of molar conductivity.
(for l/a). Whose k is 0.00276 π–1 cm –1 at 290 k.
As weak electrolytes like H2CO3, CH2COOH,
Equivalent conductivity NH4OH are ionized to a lesser extent in aqueous solution,
they have a lower value of molar conductivity.
The conductivity of all the ions produced, when one gram
equivalent of an electrolyte is dissolved in V mL of solu­ 2. Concentration of the solution: As the concentrated solu­
tion, is called equivalent conductivity. It is denoted by λV tions of strong electrolytes have significant inter­ionic
or Λv. attractions, which reduce the speed of ions, it lowers the
value of molar conductivity. As dilution decreases such
λ or Λ = k × V. attractions, it increases the value of molar conductivity.
Unit is Ω–1 cm2
9.10 Chapter 9

The limiting value of Λm (Λm∝ or the molar conduc­ 2. Dielectric constant of solvent: Higher the value of
tivity at zero concentration or at infinite dilution) can be dielectric constant of solvent, more is the value of molar
obtained by extrapolating the graph shown ahead. conductivity, as it decreases inter­ionic attractions.
3. In case of weak electrolytes, the degree of ionization 3. Viscosity of solvent:
increases, which increases the value of Λm. However, it
cannot be obtained by extrapolating the graph. The lim­ 1
Λm ∝
iting value Λm∝ for weak electrolytes can be obtained by η
Kohlrausch law.
that is, higher the value of viscosity, lower is the value of

m
molar conductivity.

Strong Electrolyte (KCl)


FA C T S TO R E M E M B E R
Besides nature of material, the magnitude of conductivity
also depends upon temperature and pressure at which the
measurements are made.
m

The conductivity of an electrolytic solution depends


upon
• Nature of the electrolyte
Weak Electrolyte (CH 3 COOH)
• Size of ions produced and their solvation (hydration)
• Nature of solvent and its viscosity
C • Concentration of the electrolytes
Figure 9.1 Effect of Dilution on Molar Conductivity • Temperature (direct relation)

Kohlrausch law NOTE


According to this law, “The equivalent conductivity of an The conductivity of metals depends upon:
electrolyte at infinite dilution (λ∝) is the sum of two values,  Nature of structure of metal
one depending only on the cation and the other only on the  Number of valence electrons per atom
anion.”  Density of metal
λ∝ or Λ∝ or Λm∝ = λc+ + λa– = (V+ λc+) + (V– λa–)
 Temperature (inverse relation)
λc+ and λa– are ionic mobility of cation and anion respec­
tively and V+, V– are no. of cations and anions performula
unit. ELECTROCHEMICAL CELL
Example: Electrochemical cell is a device in which the decrease of
It is used fo find Λm∝ for weak electrolytes for e.g., free energy during an indirect redox reaction is made to
Acetic acid. convert chemical energy into electrical energy.
• Luigi Galvani and Allesandro Volta developed
Λm (CH3COOH) = λ∝ CH3COO– + λ∝ H+
this de vice, therefore these cells are also known as
Λm∞ = Λm∞ + Λm∞ – Λm∞ Galvanic cells or Voltaic cells.
CH3COOH CH3COONa HCl NaCl Table 9.3 Electrons in a Galvanic Cell
1. Temperature: Feature Cathode Anode
Λm∝ Temperature Sign Positive due to Negative due to
As the increase in temperature decreases inter­ionic consumption of electrons release of electrons
attrations, solvation of ions, viscosity and increaseski­ Reaction Reduction Oxidation
netic energy of ions, so Λm increases with increase in Movement Into the cell Out of the cell
temperature. of electrons
Redox Reactions and Electrochemistry 9.11

Table 9.4 Difference Between Electrochemical while metal ions flow form one half cell to the other
and Electrolytic Cells through the salt bridge.
Electrochemical cell Electrolytic Cell • Here current flows from copper electrode to zinc
It is a combination of two half It is a single cell elec trode that is, cathode to anode via an external
cells, containing the same or containing the same circuit.
different electrodes in the same electrodes present in the
or different electrolytes. same electrolyte.
• Daniell cell is a reversible cell while a Voltaic cell
may be reversible or irreversible.
Anode is negative, cathode is Anode is positive,
A Voltaic cell is reversible only when it satisfies fol­
positive. cathode is negative.
lowing conditions:
Electrons move from anode to Electrons enter through
cathode in external circuit. cathode and leave 1. The emf of external source is more than that of voltaic
through anode. cell, so that current may flow from external source into
It converts chemical energy into It converts electrical the voltaic cell and cell reaction can be reversed.
electrical energy, produced as a energy into chemical
result of redox reaction. energy. Energy is 2. If emf of voltaic cell is more than that of external source,
supplied to the current flows from voltaic cell into external source.
electrolytic solution to
bring about the redox
Saltbridge
reaction.
Cell reaction is spontaneous. Cell reaction is non It maintains electrical neutrality in two com partments by
spontaneous. allowing movement of anions towards anodic compartment
Salt bridge is required. No salt bridge is and cations towards cathodic compartment.
required. • It is a glass tube having potassium chloride
Ammonium chloride ammonium nitrate in a gelatin
Ammeter form.
2e- 2e- • The gelatin allows ionic movement but prevents any
Cu
Cathode kind of mixing.
Zn
Salt Bridge • In case of potassium chloride or ammonium nitrate
Anode
2+ the ionic mobilities of cations and anions are same.
2+ Cu + 2e-
Zn
+ 1. Functions of a saltbridge

(a) A salt bridge acts as an electrical contact between the
ZnSO4 Anions CuSO4 two half cells.
(1M) Cations (1M)
(b) It prevents mechanical flow of solution, but it pro­
vides free path for the migration of ions, to maintain
an electric current through the electrolyte solution. It
Oxidation Half Cell Reduction Half Cell prevents the accumulation of charges.
(Zinc Half Cell) (Copper Half Cell) (c) A salt bridge helps in maintaining the charge balance
Figure 9.2 Daniell Cell in the two half cells.
(d) A salt bridge minimizes/eliminates the liquid junc­
tion potential.
The Daniell Cell
Liquid junction potential
A typical galvanic cell, it is designed to make use of the
spontaneous redox reaction between zinc and cupric ion to The unequal rates of migra tion of the cations and anions
produce an electric current. across a liquid­liquid junc tion give rise to a potential dif­
ference across the junction. This potential difference across
• The Daniell cell can be conventionally represented as the liquid­liquid junction is called liquid junction potential.
Zn (s) | ZnSO4 (aq) || CuSO4 (aq) | Cu(s)
• The Daniell cell reaction is represented as FA C T S TO R E M E M B E R
Zn (s) + Cu2+ (aq) Zn2+ (aq) + Cu (s)
If the salt bridge is removed, the emf of the cell drops to
• In a Daniell cell, electrons flow from zinc elec­ zero.
trode to copper electrode through external circuit,
9.12 Chapter 9

Electrodes or Half Cells Redox electrode


• An electrode and the solution in which it is dipped is This electrode includes a platinum wire dipped in a solu­
called a half cell. tion of mixture of the two salts of the same metal, but with
different oxidation states. For ex ample, Pt, Fe2+/Fe3+.
• Half cells are called anodic half cells and cathodic
half cells.
Glass Electrode
• The electrode at which oxidation takes place is called
anode. It consists of a thin walled glass bulb made out of spe­
cial types of low temperature melting glass.
• The electrode at which reduction takes place is called
cathode. Glasss electrodes are not affected by oxidizing and
reducing agents and are not easily poisoned so they
• Anode is positively charged and cathode is nega­
are finding great application in industry, in analytical
tively charged in an electrolytic cell.
chemical and biological laboratories in the measure­
• In a voltaic cell, anode has negative charge as the ment of pH.
electron will flow from it.
Glass electrode gives very good results for pH value
• In a voltaic cell, cathode has positive charge as the
ranging between O and 6. For solutions of high alkalin­
electrons are drawn towards it.
ity, high accuracy is not obtained.
• At anode de­electronation or oxidation occurs, as the
element goes into the solution as ions, due to loss of
electrons. Standard Hydrogen Electrode (SHE)
• At cathode, electronation or reduction process A hydrogen electrode in which pressure of hydrogen
occurs as ions gain electron and get discharged gas is maintained at 1 atm id he concentration of H+
here. ions in the solution is 1 M, is called a standard hydro­
gen electrode (SHE).
Types of Electrodes or Half Cells 1. The emf of a standard hydrogen electrode is taken
as 0.00 V at all temperature.
The various types of electrodes which are frequently used
in the electrochemical cells are as follows: 2. It is a reversible electrode.
3. It is used as a reference electrode.
Metal-metal ion electrode 4. A saturated calomel electrode is also used as a
It includes a metal strip dipped in the solution of its own secondary reference electrode. It has a potential of
cations. For example, –0.24 volt.
Zn/Zn2+, Cu/Cu2+, Ag/Ag+.
Electrode Potential or Single Electrode
Amalgam electrode Potential
It is similar to metal­metal ion type electrode, but here,
• Electrode potential is the tendency of an electrode to
metal is replaced by its amalgam with Hg. This is done to
accept or to loose electrons.
improve the activity of metal. For example, Zn–Hg / Zn2+
• When a metal strip M is placed in contact with the
Gas electrode solution of its own ions Mn+, an electrical potential
It comprises an inert metal foil, like platinum, immersed in difference is set up between the metal strip and the
the solution capable of furnjshing ions of gas. For example, solution due to either of the two tendencies.
Pt, H2/H+ and Pt, Cl2/Cl– Oxidation: M Mn+ + ne–
Reduction: Mn+ + ne– M
Metal-metal insoluble salt-salt anion
The electrical potential difference, between metal
This type of electrode includes metal in contact with its
strip and solution is known as half/cell electrode
sparingly solu ble salt and aqueous solution of salt contain­
potential (E).
ing same anion. For example,
• Single electrode potential cannot be measured
Hg–Hg2Cl2 (s)–KCl (aq) Calomel electrode experimentally.
Pb–PbSO4(s)–K2SO4 (aq) • The potential difference between two single elec­
trodes can be measured by using potentiometer.
Redox Reactions and Electrochemistry 9.13

• The tendency of oxidation can be represented by oxi­ E°cell or emfO = [EORP (cathode) – E°RP (anode)]
dation potential (Eop). • For cell reaction to occur the Ecell should be
• The tendency of reduction can be represented by positive. This can happen only if ERP (cathode) > ERP
reduction potential (ERP). (anode).
• Oxidation potential of an element is same in magni­ • E°cell must be positive for a spontaneous reaction.
tude but opposite in sign to its reduction potential, • The rate of an electrolytic reaction is directly propor­
that is, ERP = Eop. Therefore, by conventions, the elec­ tional to the emf of the cell.
trode potentials (E) are represented by their reduc­ • The emf of the cell depends on the intensity of the
tion potentials ERP. reaction in the cell.
• ERP values vary with the concentration of the ions • It measures free energy change for maximum
in solution at a particular temperature. The reduc­ convert ibility of heat into useful work.
tion potential of the electrode at standard state
conditions (1 molar concentration of the ions, 298 • It causes flow of current from higher value of E°
K and 1 atm pressure) is called standard reduction electrode to lower E° value electrode.
potential (E°).
• The absolute values of E° for any electrode cannot Nernst Equation
be determined, because it is impossible to work with Nernst equation gives the relationship between electrode
a single half cell, hence the E° values are determined potential and concentration of ions in the solution. It shows
with respect to standard hydrogen electrode (SHE) the dependency of electrode potential on the concentration
whose standard electrode potential (E°) is taken to be of the ions with which the electrode is reversible.
zero by convention.
• E° is an intensive property like temperature or molar Table 9.5 Difference Between emf and Cell
volume. This means that E° is same for half­cell Potential
reaction whether it is represented as Electromotive force Cell potential
2X+ + 2e– X2, It is measured by It is measured by voltmeter.
or potentiometer.
X +e– ½ X2, It is the potential difference It is the potential difference
• E° values are not thermodynamic functions and may between two electrodes between two electrodes
not be added directly. However they can be calculated when no current is flowing when current is flowing
from free energy, which is an extensive property. in the circuit. through the circuit.

The value of the electrode potential developed on an elec­ It is maximum voltage It is less than maximum
trode depends upon: obtained from cell. voltage.
• Temperature of the system. It corresponds to maximum It does not correspond
• Concentration of ions in solutions. useful work obtained from to maximum useful work
galvanic cell. obtained from galvanic cell.
• Chemical nature of the metal or non­metal.
• Number of electrons transferred in the half cell • For a single electrode involving the reduction
reaction. process,

• It does not depend on the electrode. Mn+ + ne– ne M
Nernst equation is
Electromotive Force or emf of the Cell or 2.303 RT [Mn+]
Cell Voltage E = E° + _______
nF log10_____
M
2.303 RT [M]
Electromotive fource of the cell is the potential difference E = E° – _______
nF log10 _____
[Mn+]
between the two terminals of the cell, when no current is
drawn from it. It is measured with the help of potentiome­ At 25°C or 298K
2.303 × 8.614 × 298 [Mn+]
ter or vacuum tube voltmeter. E = E° + ______________
n × 96500 log10_____
[m]
0.059
Calculating the emf of a cell E = E° + _____
n log10 [M ]/M
n+

Mathematically, it may be expressed as 0.059 [M]


Ecell or emf = [ERP (cathode) – ERP (anode)] E = E° – _____ _____
n log10 [Mn+]
9.14 Chapter 9

• Here R = Gas constant • Cu + 2AgNO3 Cu (NO3)2 + 2Ag


T = Absolute temperature The colour of the solution turns blue from colour­
E° = emf of the cell less and the rod turns from reddish brown to white.
E = Electrode potential of cell Reason: As a metal higher in the series has greater
F = Faraday number Eop or reactivity, it displaces the metal below it.
n = Number of electrons transferred • Metals above H2 can easily replace H2 from acid.
• If the electrode is solid, its active mass is taken as one. For example,
• For an electrochemical cell, having net reaction Mg + H2SO4 MgSO4 + H2
xA + yB mC + nD Oxidation Potential of Mg > oxidation Potential of H2
The emf can be calculated as R–OH + Na R–ONa + H+
x y
• Metals lower is the series cannot do so as oxidation
Ecell = E°cell +
0.059
log
[A ] [B] potential of H2 is higher than that of the following
n [C]m [D]n metals
C H A P A
In using the above equation, the following facts
should be kept in mind. Cu Hg Ag Pt Au
• Concentration or activity of gases is expressed in Cu + H2SO4 XX (No reaction in ordinary
terms of their partial pressures. conditions)
• The number of electrons transferred n should be cal­ • Oxides of lower metals (C H A P A) are easily
culated from the balanced net cell reaction. reduced by H2 or carbon.
• Concentration or activity of solids is taken to be • A higher value of oxidation potential implies that the
unity. substance undergoes oxidation easily; this means that
• Relationship between free energy change (ΔG) and such a substance is a good reducing agent. Similarly,
cell potential (E) can be written as a higher value of reduction potential implies that the
substance undergoes reduction easily and hence is a
ΔG= – nF Ecell good oxidizing agent.
For standard state conditions, • Metals above hydrogen in the electrochemical series
ΔG°= – nF E°cell have positive oxidation potential, that is, they can act
• Equilibrium constant (Keq) of net cell reaction is as reducing agents.
related to the standard emf as Reducing property α EOP
0.059 For example, Li is the strongest reducing agent due
E°cell = ____
n log10 Keq to highest value of oxidation potential.
ELECTROCHEMICAL SERIES For example, if for A, EOP is –0.44 V and for B,
EOP is –0.28 V, then A is a better reducing agent
Electrochemical series is the arrangement of various elec­ than B.
trodes or elements in the increasing order of their standard • Metals [C H A P A] have high ERP, so these are oxi­
reduction potential or decreasing order of their standard dizing agents.
oxidation potential.
Oxidizing power α ERP
Features of Electrochemical Series F2 > Cl2 > Br2 > I2
• A metal lower in the series is replaced by ones above Oxidation Potential decreases
it while the one higher in the series is coated by the
HI > HBr > HCl > HF
metal below it. For example,
Reduction Potential decreases
Zn + CuSO4 ZnSO4, + Cu
As EOP of I– > Br– > Cl– > F–
• Zn is coated by Cu, Zn–Cu couple is coated by Cu.
• Elements with more positive ERP are discharged
• Here solution turns blue to colourless and the colour before the ones with less positive ERP, Discharging
of the rod changes from grey white to reddish brown. tendency increases from Li to F (for M+)
Redox Reactions and Electrochemistry 9.15

Table 9.6 Electrochemical Series


Reaction (oxidized form) Reduced form E°/V

Li+ + e– Li(s) –3.05


+ –
K +e K(s) –2.93
Ca2+ + 2e– Ca(s) –2.87
Na+ + e– Na(s) –2.71
Mg2+ + 2e– Mg(s) –2.36
Al3+ + 3e– Al(s) –1.66
Zn2+ + 2e– Zn(s) –0.76
Cr3+ + 3e– Cr(s) –0.74
Fe2+ + 2e– Fe(s) –0.44
2+ –
Ni + 2e Ni(s) –0.25
Sn2+ + 2e– Sn(s) –0.14
Pb2+ + 2e– Pb(s) –0.13
2H+ + 2e– H2(g) 0.00
AgBr (s) + e– Ag (s) + Br– 0.10
AgCl (s) + e– Ag(s) + Cl– 0.22
Cu2+ + 2e– Cu(s) 0.34
Cu+ + e– Cu(s) 0.52
I2 + 2e– 2I– 0.54
O2 (g) + 2H+ + 2e– H2O2 0.68
Fe3+ + e– Fe2+ 0.77
Ag+ + e– Ag(s) 0.80
2Hg2+ + 2e– Hg22+ 0.92
NO3– + 4H+ + 3e– NO(g) + 2H2O 0.97

Br2 + 2e– 2Br– 1.09


MnO2 (s) + 4H+ + 2e– Mn2+ + 2H2O 1.23
O2(g) + 4H+ + 4e– 2H2O 1.23
Cr2O72– + 14H+ + 6e– 2Cr3+ + 7H2O 1.33
Cl2(g) + 2e– 2Cl– 1.36
Au3+ + 3e– Au(s) 1.40
MnO4– + 8H+ + 5e– Mn2– + 4H2O 1.51
H2O2 + 2H+ + 2e– 2H2O 1.78
Co3+ + e– Co2+ 1.81
F2(g) + 2e– 2F– 2.87
Decreasing strength of oxidizing agent Decreasing strength of reducing agent

• In case of anion, hydroxides of metals lower in the series are weakly


Discharging order ∝ _________1
Value or ERP
acidic and their salts undergo hydrolysis.
SO–2 < NO–3 < OH– < Cl– < Br– < I–
4
CONCENTRATION CELLS
• Reactivity of metal α EOP
Therefore, Li is more reactive than Mg. • Concentration cell is one in which emf arises as a
• Reactivity of non­metal α ERP result of different concentrations of the same electro­
lyte in the component half cells.
Therefore, the order of reactivity is F2> Cl2 > Br2 > I2
• The two solutions are connected by a salt bridge and
• Hydroxides of metals higher in the series are strongly
the electrodes are joined by a piece of metallic wire.
basic and their salts do not undergo hydrolysis, while
9.16 Chapter 9

• Reduction occurs in the concentrated compartment overall reaction does not involve any ion, whose con­
while oxidation occurs in the diluted compartment. centration can change over time.
2.303 RT [C ] • It is used in hearing aids and watches.
E = E° – _______
nF log ___
2
[C1]
or Secondary Cells
2.303 RT [C1]
E = E° + _______
nF log ___
[C2] Secondary cells can be recharged by passing current so that
they can be used again, as electrode reactions are revers­
ible. For example, lead storage battery and Ni–Cd storage
SOME COMMERCIAL CELLS cell.
Primary Cells
Lead storage battery
• In primary cells redox reaction occur only once, so • Lead storage battery is used in automobiles.
they cannot be recharged.
• It contains six voltaic cells in a row.
• The cell becomes dead after some time as electrode
• Anode is made up of spongy lead and cathode is
reactions cannot be reversed. For example, dry cell
made up of a grid or lead packed with PbO2
and mercury cell.
• Electrolyte used is an aqueous solution of H2SO4
Dry cell (38 % by mass)
• Dry cell is a compact form of Lechlanche cell. • Cell reactions:
• It uses a Zn­container as the anode and the cathode At anode
is made up of a graphite rod surrounded by MnO2 + Pb + S O –2
4
PbSO4 + 2e–
carbon. At cathode
• Here, a paste of NH4Cl and ZnCl2 is filled in between PbO2 + SO–2 + 4H+ + 2SO–2 PbSO4 + 2H2O
4 4
the electrodes. The overall reaction is
• Cell Reactions:
Pb + PbO2 + 4H+ + 2SO–2 4
2PbSO4 + 2H2O
At anode
• During discharge of this battery H2SO4 is consumed
Zn (s) Zn+2 (aq) + 2e– and its concentration falls to 1.2 g/ml.
At cathode Nickel-Cadmium cell A rechargeable nickel­cadmium
2MnO2 (s) + 2N H +4 (aq) + 2e– cell has a jelly roll arrangement and is separated by a layer
Mn2O3 (s) + 2NH3 (g) + H2O soaked in moist sodium or potassium hydroxide.
• Zn+2 combines with NH3 to form diammine Zn(II) Fuel cell
cation.
• The best example of fuel cell is hydrogen­oxygen
• Dry cell has a short life, as NH4Cl (acidic) corrodes
fuel cell and its cell potential is 1 volt.
the Zn­container even when the cell is not in use.
• The cell potential is 1.25 to 1.5 volt 2H2 + O2 2H2O
• Spaceships use hydrogen fuel cell while in space.
Mercury cell • It is a pollution free, continuous source of energy and
• In the commonly used mercury cell, the reducing agent is highly efficient.
is zinc and the oxidizing agent is mercury (II) oxide. Cell Reactions:
• Cell Reactions: At anode
At anode 2H2 (g) + 4OH– (aq) 4H2O (l) + 4e–
Zn (Hg) + 2OH– ZnO (s) + H2O + 2e– At cathode
At cathode O2 (g) + 2H2O (l) + 4e– 4OH– (aq)
HgO + H2O + 2e– Hg(l) + 2OH– Net reaction is
The overall reaction is 2H2 (g) + O2 (g) 2H2O (l)
Zn(Hg) + HgO (s) ZnO (s) + Hg(l) • Requirement: It should be compact, light, rugged and
• The cell potential is approximately 1.35V and should not undergo much volume drop during use.
remains constant throughout the cells life, as the
Redox Reactions and Electrochemistry 9.17

RUSTING OR CORROSION OF METAL galvanized iron sheets lose their lusture and protects the iron
from further corrosion.
Rusting of metal is the destruction of metal on reaction Iron can be coated with copper or tin. If the coating is
with oxygen present in the environment. It is the reverse broken, iron is exposed and being more reactive than iron,
of extraction of metals and is a redox process. Chemically copper and tin corrode more rapidly.
iron rust is Fe2O3. xH2O
2Fe + O2 + 2H2O 2Fe(OH)2 Electrical protection
Fe(OH)2 dehydrates and oxidizes to FeO and Fe(OH)3, then
or cathodic protection More electropositive metals like Zn,
finally into rust by dehydration.
Mg or Al may be coated on iron pipes buried in the moist
• An electrochemical cell known as corrosion cell is
soil, canals, storage tanks.
developed at the surface of iron.
Anode—Pure iron Using antirust solutions
Cathode—impure surface
These are alkaline phosphate and alkaline chromate solu­
Electrolyte tions. The alkaline nature of solutions prevent availability
CO2 + H2O H2CO3 H+ + HC O–3 of H+ ions. On the surface of iron, a protective, insoluble
SO2 + H2O H2SO3 H+ + HSO–3 thin film of iron phosphate is formed. These are used in car
Anode reaction radiators to prevent rusting.
Fe Fe2+ + 2e–
Cathode reaction SOLVED EXAMPLES
2H+ + 2e– 2H
2H + ½ O2 H2O
Conductance
The over all reaction is
Fe + 2H+ + ½ O2 Fe2+ + H2O 1. 0.05 M NaOH solution offered a resistance of 31.16 Ω in
a conductivity cell at 298 K. If cell constant is 0.367 cm–1.
At surface
Find out the molar conductivity of NaOH solution.
2Fe2+ + ½ O2 + 2H2O Fe2O3 + 4H+
Fe2O3 + xH2O Fe2O–3xH2O Solution
Cell constant
Conductivity (k) = __________
R
The overall reaction at surface is
0.367
2Fe2+ + ½O2 + (x + 2)H2O Fe2O3xH2O + 4H+ = _____
31.16 = 0.0118 cm
–1

Rust Λm = _______
0.0118 × 1000
M = ___________
k × 1000
0.05
• Reactivity of metal, presence of impurities, presence
of air and moisture, strains in metal and presence of = 236 S cm2 mol–1
electrolyte are the factors promoting corrosion.
2. A salt solution of 0.30 N placed in a cell whose elec­
trodes are 1.9 cm apart and 3.6 cm2 in area offers a
Prevention of Corrosion resistance of 20 Ω. Calculate equivalent conductivity
Sacrifical protection of solution.
Covering the surface of iron with a layer of metal which Solution
is more active than iron, prevents the iron from losing R = 20 Ω, l = 1.9 cm
electrons (plating metal with a thin layer of less easily oxi­
dizable metal like Mg, Zn, Sn). a = 3.6 cm2 , N = 0.30 N
Cell constant = _l = ___ = 0.528 cm–1
1.9
a 3.6
Galvanization
Specific conductance (K)
Covering iron with more active metals like zinc. The layer of = Conductance × Cell constant
Zn on the iron surface when comes in contact with moisture,
= (1/R) × cell constant
oxygen and CO2 in air, forms a protective, invisible, thin layer
of basic zinc carbonate (ZnCO3.Zn(OH)2) due to which the = (1/20) × 0.528
= 0.0264 S cm–1
9.18 Chapter 9

0.0264 × 1000
K × 1000 = ___________
Λeq = _______
0.30 7. A current of 3.7 amperes is passed for 360
N
= 88 S cm eq .
2 –1 minutes between nickel electrodes of a 2M solution of
Ni(NO3)2. What will be the molarity of the solution at
3. The molar conductivities of CH3COOH at 25°C at the end of the electrolysis? (At. wt of Ni = 58.70)
the concentration of 0.1 M and 0.001 M are 5.20 and
Solution
49.2 cm2 mol–1 respectively. Calculate the degree of
dissociation of CH3COOH at these concentrations. Ni2+ + 2e– Ni (s)
E. I. t
(Λ∝m (CH3COOH) = 390.7 S cm2 mol–1) w = ____
F
58.70 × 3.7 ×360 × 60
________________
Solution = 96500 × 2 = 24.307 g
Λ c
Number of moles of Ni deposited
Degree of dissociation ( α ) = m

24.307
Λ m = _____
58.70 = 0.414 mol
5.20
= _____
390.7 = 0.013 = 1.3% Molarity of remaining solution = 2.0 – 0.414
49.2
0.001 M = _____
390.7 = 0.125 = 12.5% = 1.586 M

4. If Λ∝m of HCl, NaCl and CH3COONa are 425, 128 and 8. Electrolysis of a solution of MnSO4 in aqueous sul­
96 Ω–1 cm2 mol–1 respectively, calculate the value of phuric acid is a method for the preparation of MnO2 as
Λ∝m for acetic acid. per the reaction
Solution Mn2+(aq)+2H2O MnO2(s) + 2H+(aq) + H2(g)
Λ∝m CH COOH = Λ∝m CH COONa + Λ∝m HCl – Λ∝m NaCl
3 3
Passing a current of 27 A for 24 hours gives one kg of
= 96.0 +425.0 – 128.0 MnO2. What is the value of current efficiency?
= 393.0 Ω–1 cm2 mol–1
Solution
E.I.t
Faraday’s Law Wt. of MnO2 (w) = ____
F
87 × I × 24 × 60 × 60
5. How many hours does it take to reduce 3 mol of Fe3+ to 1000 = _______________
2 × 96500
Fe2+ with 2.0 ampere current? (F= 96500 C mol–1). I = 25.67 ampere
25.67 ×100
Solution Current efficiency = ________
27 = 95%
Fe3++ e– Fe2+
Charge 9. How many grams of silver can be plated out on a serving
3 × 96500
t = ______ ________
Current = 2 tray by electrolysis of a solution containing silver in +1
oxidation state for a period of 8.0 hours at a current of
= 144750 second = 40.2 hours
8.46 amperes? What is the area of the tray if the thickness
6. Calculate the number of coulombs required to deposit of the silver plating is 0.00254 cm? Density of silver is
5.4 g of Al when the electrode reaction is 10.5 g/cm3.
Solution
Al3+ + 3e– Al
Ag+ + Ie– Ag (s)
Given (Atomic mass of Al = 27 g mol–1, F = 96500 C E.I.t
mol–1) Wt. of Ag = ____
F
107.9 × 8.46 × 8 × 60 × 60
___________________
Solution = 96500 = 272.43g
Al3+ + 3e– Al wt of Ag 272.43
V = _______
density = _____
10.5 25.64 mL
For 1 mol (27.0 g) of Al requires = 3 × 96500 C
3 × 96500 Volume 25.94
1 g of A1 requires = ________
27.0 Area of tray = ________ ______
Thickness = 0.00254
3 × 96500 × 5.4
5.4 g of A1 requires = ____________
27.0
= 10212.6 cm2
= 57900 C
Redox Reactions and Electrochemistry 9.19

10. Calculate the quantity of electricity required to reduce 13. Calculate the cell potential for the cell containing 0.10
6.15 g of nitrobenzene to aniline if the current effi­ M Ag+ and 4.0 M Cu2+ at 298 K.
ciency is 68 %. If potential drops across the cell is 7.0
volts. Calculate the energy consumed in the process. Given E°Ag+/Ag = 0.80 V

Solution E°Cu2+/Cu = 0.34 V


C6H5NO2 + 6H+ + 6e– C6H5NH2 + 2H2O Solution
Mol.wt
Eq. wt of nitrobenzene = ______ 6 Cu (s) | Cu2+ (4.0 M) || Ag+ (0.1 M) | Ag (s)
123
= ___
6 = 20.05
Here n = 2;
E.I.t. 20.5 × I × t × 68 0.0591 [Ag+ 2
]
_____ ____________
w = 96500 = 100 × 96500 = 6.15 Ecell = E°cell + _____
n log
_____
[Zn2+]
[As I = 68 / 100]
0.0591
= 0.80 – (– 0.76) + _____
2 log10______
[0.01]2
[0.1]
I × t = 42573.5 Coulombs
= 0.46 + 0.0295 × log10 2.5 × 10–3
Energy consumed = Q × V
= 42573.5 × 7.0 = 298.015 kJ = 0.46 – 0.07676 = 0.3832 V

emf and Nernst Equation 14. For the galvanic cell

Cr (s) | Cr3+ (2.23 × 10–7 M) || Ag+ (0.0264 M) | Ag (s)


11. Calculate the cell potential of the given cell at 25°C.
at 25°C
(R = 8.31 J K–1 mor–1, F = 96500 C mol–1).
Calculate the emf of the cell.
Ni (s) | Ni2+ (0.01 M) || Cu2+ (0.1 M) | Cu(s)
Given: E° (Ag+/Ag) = 0.80 V
Given: E°Cu2+/Cu = + 0.34 V
E° (Cr+3 / Cr) = – 0.74 V
E° (Ni2+/Ni) = – 0.25 V
Solution
Solution 0.0591 [Ag+]3
2.303 RT [Cu2+] Ecell = E°cell + _____ _____
n log [Cr+3]
Ecell = E°cell + _______
nF log10 _____
[Ni2+]
0.0591 [0.064]3
2.303 × 8.31 × 298
= 0.80 – (0.74) + _____
3 log10 _________
[2.23 × 10–7]
= 0.34 – (–0.25) + ______________
2 × 96500 0.1
log10 ____
0.01 = 1.54 + 0.0197 × 1.9141
0.0591
= 0.59 + _____
2 log10 10 = 0.6195 V
= 1.577

12. Calculate the emf of the following cell at 298 K. 15. Calculate the equilibrium constant for the reaction
Zn | Zn (0.1 M) || Ag (0.01 M) | Ag
2+ +
Fe2+ + Ce4+ Fe3+ + Ce3+.
Given: E°Zn2+/Zn = – 0.76 C Given: E° (Ce+4 / Ce+3) = 1.44 V
E°Ag+/Ag = +0.80 V E° (Fe+3/Fe+2) = 0.68 V
Solution
Solution
[Ag+]2
0.0591
Ecell = E°cell + _____ _____
n log [Zn2+]
n
log Kc = _____
0.0591 × E°cell
0.0591 [0.01]2 1
= _____
0.0591 × 0.76 = 12.8813
= 0.80 – (–0.76) + _____
2 log10 ______
[0.1] Kc = antilog of 12.8813
0.0591
= 1.56 + _____
2 log10 1 × 10–3 Kc = 7.608 × 1012
= 1.56 – 0.08865 = 1.471 V
9.20 Chapter 9

,
CONCEPTS AT A GLANCE

Auto Oxidation Potential Gradient


 The formation of H2O2 by the oxidation of H2O is  Potential difference applied at the electrodes divided
called as auto oxidation. by the distance between the electrodes is known as
 Here the substance which activates oxygen is called potential gradient.
an activator. For example P, Pb, turpentine.  Although standard oxidation potential of Cl– ion (–
As in, 1.36 V) is lower than that of water (–1.23 V), still it
Pb + O2 PbO2 is the Cl– which is oxidized to Cl2 at the anode during
PbO2 + H2O PbO + H2O2 electrolysis of an aqueous solution of NaCl. This is
due to over voltage, that is, water needs greater volt-
Induced Oxidation age for oxidation to form O2, than that needed for
oxidation of Cl to Cl2. This extra voltage required is
 Here a substance gets oxidized in presence of
called over voltage.
another substance. °
Ecell – Ereference
For example, Na2AsO3 (sodium arsenite) gets oxidized  pH = __________
0.0591
in presence of sodium sulphite as follows
In this case, the reference half cell is a calomel electrode.
Na2AsO3 + O2 XX (No Reaction)
 The thermodynamic efficiency of fuel cells is the ratio of
Na2SO3 + O2 Na2SO5 the electrical free energy to the ∆H of the reaction.
Na2SO + Na2AsO3 Na2AsO4 + Na2SO4 ∆G
___
η = ∆H
The over all reaction is
Na2SO3 + NaAsO3 + O2 Na2SO4 + Na2AsO4 Hitroff Number
Sodium
arsenate Current carried by ion
t or n =
Spectator Lons Total current
 It is the species which is present in the solution, but tc + ta = 1
does not change in a chemical reaction and is omit- IαV
ted while writing the net ionic reaction.
Vc
For example, tc =
Vc + Va
Zn + 2H+ + 2Cl– Zn2+ + 2Cl– + H2
Here Cl– ions are spectator ions.
,
Redox Reactions and Electrochemistry 9.21

TOOL BOX AND SUMMARY OF IMPORTANT RELATIONS

Faraday’s first Law of Electrolysis Kohlrausch Law


w = Z. I. t λ∝ or Λ∝ = λ+c +λ–a
M
___
Z = nF λe+ + λa– = ionic mobility of cation act anion
Eq. wt = Z × 96500
Cell Voltage or emf of a Cell
w
__ Q
_____
E = 96500  Mathematically, it may be expressed as
Ecell or emf = [Ered (cathode) – Ered (anode)]
Faraday’s Second Law of Electrolysis
= [Ered (Cathode) – Eoxi (anode)]
E1 M1 Z1 .I.t Z1 º º º
= = = Ecell or emfº = [Ered (cathode) – Ered (anode)]
E 2 M 2 Z 2 .I.t Z 2
Current Efficiency Nernst Equation
0.059
_____ [Mn+]
____
Desired extent
_____________________ E = E° + n log [M]
C.E. = Theoretical extent of reaction
× 100
 Relationship between free energy change and cell potential
Ohm’s Law can be written as
E
__ ∆G = – nF Ecell
I= R
ρ__l
__ for standard state conditions,
R α l/a or R= a °
∆G° = – nF Ecell
ρ = Specific resistance  Equilibrium constant of net cell reaction is related to the
standard emf as
Specific Conductance (k, Kappa)
0.059
_____
k = 1__ρ = 1__ _l
R = a
E°cell = n log10 K.

E cell − E reference
Equivalent Conductivity  pH =
0.0591
λv or λΛ
 Thermodynamic efficiency of fuel cells
λ or Λ = k × V ∆G
___
η = ∆H
w
__ NV
____
E = 1000  Relation between E° of half cells having a metal in different
K × 1000 oxidation states
λ=
N ∆G3° = ∆G1° + ∆G2°
Molar Conductivity – n3 F E3° = –n1 F E1° – n2 F E2°

1000 × k × V n3 E3° = n1 E1° + n2 E2°


λ m or Λ v = E1° + n2E2°
n1__________
N E3 = n 3
9.22 Chapter 9

EXERCISES

Single Option Correct Type

1. Which of the following is a redox reaction? 8. In electrolyses of NaCl, when Pt electrode is taken
(a) NaCl + KNO3 NaNO3 + KCl then H2 is liberated at cathode, while with Hg cathode
(b) CaC2O4 + 2HCl CaCl2 + H2C2O4 it forms sodium amalgam because
(c) Mg(OH)2 + 2NH4Cl MgCl2 + 2NH4OH (a) Hg is more inert than Pt
(b) more voltage is required to reduce H+ at Hg than at
(d) Zn + 2AgCN 2Ag + Zn(CN)2
Pt
2. Which of the following is not a redox reaction? (c) Na is dissolved in Hg while it does not dissolve
(a) MgCO3 MgO + CO2 in Pt
(d) concentration of H+ ions is larger when Pt elec­
(b) O2 + 2H2 2H2O
trode is taken.
(c) Na + H2O NaOH + ½ H2
(d) SnCl4 SnCl2 + Cl2 9. In the silver plating of copper, K[Ag(CN)2] is used
instead of AgNO3. The reason is
3. In the following rection (a) a thin layer of Ag is formed on Cu
4P + 3KOH + 3H2O 3KH2PO2 + PH3 (b) more voltage is required
(a) P is only oxidized (c) Ag+ ions are completely removed from solution
(b) P is only reduced (d) less availability of Ag+ ions, as Cu cannot displace
(c) P is both oxidized as well as reduced Ag from [Ag(CN)2–] ion
(d) none of these 10. Zn gives H2 gas with H2SO4 and HCl but not with
HNO3 because
4. The oxidation number of V in Cs4Na(HV10O28) is
(a) Z
n acts as oxidizing agent when reacts with HNO3
(a) +2 (b) +5
(b) HNO3 is weaker acid then H2SO4 and HCl
(c) –2 (d) +3
(c) in electrochemical series Zn is above hydrogen
5. The most convenient method to protect the bottom of (d) N O –3 is reduced in preference to hydronium ion
ship made of iron is
(a) coating it with red lead oxide 11. The oxidation state of oxygen in O2PtF6 is
(b) white tin plating (a) zero (b) – ½
(c) connecting it with Mg block (c) + ½ (d) +l
(d) connecting it with Pb block 12. The oxidation number of phosphorous in ATP (adenosine
6. Given E° (Fe / Fe) = – 0.44 V and E° (Fe / Fe ) =
2+ 3+ 2+ triphosphate) is
0.77 V (a) 2 (b) 3
(c) 4 (d) 5
Fe2+, Fe3+ and Fe are placed together then
(a) Fe3+ increases 13. In the reaction,
(b) Fe3+ decreases
I2 + 2S2O2–3
2I– + S4O2–
6
, equivalent weight of
(c) [Fe3+]/[Fe2+] remains unchanged iodine will be equal to
(d) Fe2+ decreases (a) twice its molecular weight
7. Which reaction is not feasible? (b) its molecular weight
(a) 2KI + Br2 2KBr + I2 (c) ½ its moleculear weight
(b) 2KBr + I2 2KI + Br2 (d) ¼ its molecular weight
(c) 2KBr + Cl2 2KCl + Br2
(d) 2H2O + 2F2 4HF + O2
Redox Reactions and Electrochemistry 9.23

14. Removal to basic oxide from metal before electro­ (a) 0 and –l (b) –l and –2
plating is known as (c) –2 and 0 (d) –2 and +1
(a) poling (b) galvanizing 22. The brown ring complex compound is formulated as
(c) pickling (d) anodizing [Fe(H2O)5NO]SO4. The oxidation state of iron in the
15. Among the following molecules, in which does compound is
bromine show the highest oxidation number? (a) 1 ` (b) 2
(a) Hg2(BrO3)2 (b) Br–Cl (c) 3 (d) 0
(c) KBrO4 (d) Br2 23. A metal ion M3+ loses three electrons to produce
another cation. The oxidation number of the metal in
16. The equivalent weight of Na2S2O3 in the reac­
the cation is
tion 2Na2S2O3 + I2 Na2S4O6 + 2NaI is
(M = molecular weight) (a) +3 (b) +4
(a) M (b) M/4 (c) –3 (d) +6
(c) M/0.5 (d) M/2 24. A, B and C have the oxidation numbers of +6, –2 and
17. The equivalent weights of KMnO4 in an acidic, a –1 respectively, the possible molecular formula when
neutral and a strong alkaline medium respectively are these atoms combine will be
(M = molecular weight) (a) A2BC (b) AB2C2
(a) M/5, M/2, M (b) M/5, M/3, M/2 (c) ABC2 (d) AB2C
(c) M/5, M/3, M (d) M/3, M, M/5
25. Oxidation numbers of carbon in H2C2O4, CH4 and dia­
18. For the redox reaction mond respectively are
(a) +3, 4 and +4
MnO–4 + C2O2– 4
+ H+ Mn2+ + CO2 + H2O, the
correct coefficients of the reactants for the balanced (b) +3, – 4 and zero
reaction are (c) +6, +4 and zero
(d) +6 , +4 and +4
MnO–4 C2O2–
4
H+
(a) 2 5 16 26. Which of the following does not represent redox re actions?
(b) 16 5 2 (a) Cr2O2– 7
+ 2OH– Cr O 2–
4
+ H2O
(c) 5 16 2 (b) S O 2–
4
+ 2I –
+ 2H +
I 2
+ S O2–
4
+ H2O
(d) 2 16 5 (c) 2Ca(OH)2 + 2Cl2 Ca(ClO)2
+ CaCl2 + 2H2O
19. When white phosphorus reacts with caustic soda, the (d) PCl5 PCl3 + Cl2
products are PH3 and NaH2PO2. This reaction is an
example of 27. If Cl2 is passed through hot NaOH, oxidation number
(a) oxidation of Cl changes from
(b) reduction (a) –1 to 0 (b) 0 to –1
(c) disproportionation (c) 0 to +5 (d) 0 to +l
(d) neutralization
28. Cr2O2–
7
+ 6I– + 14H+ 2Cr3+ + 7H2O + 3I2
20. In the following reaction Equivalent weight of Cr2O7 is
2–

(a) M/3 (b) M/6


Cr(OH)3 + OH– + I O –3 Cr O 2– + H2O + I–
4
(c) M/2 (d) none of these
(a) IO3– is oxidizing agent
(b) Cr(OH)3 is oxidized 29. The cell constant of a solution, whose specific conduc­
tance and observed conductance are same, is equal to
(c) 6e– are being taken per 1 atom
(a) 1 (b) 0
(d) all are true
(c) 10 (d) 100
21. The oxidation states of the most electronegative ele­
30. Electrolysis of dilute H2SO4 will give
ment in the products of the reaction of BaO2 with
dilute H2SO4 are (a) SO2 and O2 (b) H2 and O2
(c) H2O and H2 (d) SO2 and H2
9.24 Chapter 9

31. In the electrolysis of a fused salt, the weight of the 40. Aqueous copper sulphate solution is electrolyzed using
deposit on the electrode, will not depend on platinum electrodes. The electrode reaction occurring
(a) current intensity at cathode is
(b) electrochemical equivalent of the ions (a) Cu2+ (aq) + 2e– Cu (s)
(c) time for electrolysis (b) Cu (s) Cu2+ (aq) + 2e–
(d) temperature of the bath (c) 2H2O (l) O2 (g) + 4H+ (aq) + 4e–
(d) O2 (g) + 4H (aq) + 4e–
+
2H2O (l)
32. On electrolysis of acidified water, if volume of hydro­
gen liberated is 5.6 cm3, then the volume of oxygen 41. The variation of ____________
equivalent conductance of weak
liberated equal to electrolyte with √ concentration is correctly shown in
(a) 1.4 cm3 (b) 2.8 cm3 figure
(c) 8.2 cm
3
(d) 5.6 cm3 (a) (b) Y
Y
33. By removing a salt bridge between the two half cells,
the voltage
(a) increase gradually (b) increase rapidly
V V
(c) does not change (d) drops to zero
X X
C
34. The best conductor of electricity is a 0.1 M solution of C
(a) H2SO4 (b) CH3COOH
(c) Y (d) Y
(c) CH3CH2COOH (d) boric acid
35. For reducing 1 mol of Cr2O2–
7
to Cr3+, the charge
required is V V
(a) 3 × 96500 coulomb
X X
(b) 6 × 96500 coulomb C C
(c) 0.3 F
(d) 0.6 F 42. Molten CuCl2 is electrolyzed using platinum elec­
trodes. The reaction occurring at anode is
36. Which of the following reactions is used to make a (a) 2Cl– Cl2 (g) + 2e–
fuel cell?
(b) Cl2 (g) + 2e –
2Cl–
(a) Cd(s) + 2Ni(OH)3 (s) CdO(s)
(c) Cu2+ + 2e– Cu (s)
+ 2Ni(OH)2 (s) + H2O (l)
(d) Cu(s) Cu + 2e–
2+
(b) Pb(s) + PbO2 (s) 2H2SO4 (aq)
2PbSO4 (s) + 2H2O (s) 43. What is the effect of dilution on the equivalent conduc­
(c) 2H2 (g) + O2 (g) 2H2O (l) tance of strong electrolyte?
(d) 2Fe (s) + O2 (g) + 4H+ 2Fe2+ (aq) (a) decrease on dilution(b) remains unchanged
+ 2H2O (l) (c) increase on dilution (d) none of these

37. When a Zn strip is placed in CuSO4 solution, Cu gets 44. When an acid cell is charged, then
precipitated, because standard oxidation potential of (a) voltage of cell increases
Zn is (b) electrolyte of cell dilutes
(a) < Cu (b) > Cu (c) resistance of cell increases
(c) < SO4 (d) > SO4 (d) none of these
38. Laws of electrolysis was given by 45. Which of the following is not correct?
(a) Faraday (b) Ostwald (a) aqueous solution of NaCl is an electrolyte
(c) Arrhenius (d) Lamark (b) t he units of electrochemical equivalent are g. coulomb
(c) in the Nernst equation, ‘n’ represents the number
39. In an aqueous solution, hydrogen will not reduce of electrons transferred in the electrode reaction
(a) Ag+ (b) Cu2+ (d) standard reduction potential of hydrogen electrode
(c) Zn
2+
(d) Fe3+ is zero volts
Redox Reactions and Electrochemistry 9.25

46. What is the electrochemical equivalent (in g cou­ 53. The standard reduction potential values of three
lomb–1) of silver? metallic cations X, Y, and Z are 0.52, –3.03 and –1.18
(a) 108F respectively. The order of reducing power of the corre­
(b) 108/F sponding metal is
(c) F/108 (a) Y>Z>X (b) X>Y>Z
(d) 1/108 F (c) Z>Y>X (d) Z>X>Y

47. Which one of the following condition will increase 54. Among the following, identify the species with an
the voltage of the cell represented by the equation atom in +6 oxidation state
Cu (s) + 2Ag+ (aq) Cu2+ (aq) + 2Ag (s)? (a) Mn O –4 (b) Cr(CN )3–

6

(a) increase in the dimension of Cu electrode (c) Ni F 6


2–
(d) CrO2Cl2
(b) increase in the dimension of Ag electrode 55. Variation of resistance with increase in cell constant
(c) increase in the concentration of Cu2+ ions gives graph of the type
(d) increase in the concentration of Ag+ ions (a) Y (b) Y
48. During an electrochemical process,
(a) Gibbs free energy increases
(b) Gibbs free energy remains constant
R R
(c) n o prediction can be made about Gibbs free energy
(d) Gibbs free energy decreases X X
l l
49. The equivalent conductance at infinite dilution of a a
HCl and NaCl are 426.15 and 126.15 mho cm2 g eq–1
respectively. It can be said that the mobility of
(c) Y (d) Y
(a) H+ ions is much more than that of Cl– ions
(b) Cl– ions is much more than that of H+ ions
(c) H+ ions is much more than that of Na+ ions R R
(d) Na+ ions is much more than that of H+ ions
X X
50. Saturated solution of KNO3 is used to make ‘salt l l
bridge’ because a a
(a) velocity of K+ is greater than that of NO–3
56. A standard hydrogen electrode has zero electrode
(b) velocity of N O–3 is greater than that of K+
potential because
(c) velocities of both K+ and NO–3 are nearly the same
(a) hydrogen is easiest to oxidize
(d) KNO3 is highly soluble in water
(b) its electrode potential is assumed to be zero
51. The reaction, (c) hydrogen atom has only one electron
3ClO– (aq) ClO3– (aq) + 2Cl– (aq) (d) hydrogen is the lightest element
is an example of 57. In the reaction,
(a) oxidation reaction
3Br2 + 6C O2–
+ 3H2O 5Br– +Br O–3 + 6HC O–3
(b) reduction reaction 3

(c) disproportionation reaction (a) bromine is oxidized and carbonate is reduced


(d) decomposition reaction (b) bromine is reduced and water is oxidized
(c) bromine is neither reduced nor oxidized
52. The correct order of equivalent conductance at infinite
(d) bromine is both reduced and oxidized
dilution of LiCl, NaCl and KCl is
(a) LiCl > NaCl > KCl 58. Of the following metals, those that cannot be obtained by
(b) KCl > NaCl > LiCl the electrolysis of the aqueous solution of their salts are
(c) NaCl >KCl> LiCl (a) Ag and Mg (b) Ag and Al
(d) LiCl > KCl > NaCl (c) Mg and Al (d) Cu and Cr
9.26 Chapter 9

59. When a copper wire is placed in a solution of AgNO3, 65. Which one of the following graphs correctly represents
the solution acquires blue colour. This is due to the the change in electrical conductivity as copper (II)
formation of carbonate is gradually added to dilute sulphuric acid
(a) Cu+ ions (a) (b)

conductivity

conductivity
(b) Cu2+ ions

Electrical

Electrical
(c) soluble complex of copper with AgNO3
(d) Cu– ion by the reduction of Cu
Mass of Mass of
60. The electric charge for electrode deposition of one CuCO3 added CuCO3 added
gram equivalent of a substance is (c) (d)

conductivity
(a) charge on one mole of electrons

conductivity
Electrical

Electrical
(b) 96500 couloumbs per second
(c) one ampere per second
(d) one ampere for one hour Mass of Mass of
CuCO3 added CuCO3 added
61. The Nernst equation, E = E° – (RT / nF) In Q indi­
cates that the equilibrium constant Kc will be equal to 66. Consider the following four electrodes
Q when (1) Cu2+(0.0001 M) | Cu(s)
(a) E = zero (b) E°=1 (2) Cu2+(0.1M)|Cu(s)
(c) E = E° (d) RT/nF=1 (3) Cu2+(0.01M)|Cu(s)
62. In the reaction (4) Cu2+(0.001 M)|Cu(s)

4Fe + 3O2 4Fe3+ + 6O2–


2
, If the standard reduction potential of Cu2+ | Cu is +0.34
which of the following statements is incorrect? V, the reduction potentials (in volts) of the above elec­
trodes following the order
(a) metallic iron is reducing agent
(a) 1 > 4 > 3 > 2 (b) 1 > 2 > 3 > 4
(b) Fe3+ is an oxidizing agent
(c) metallic iron is reduced to Fe3+ (c) 3 > 4 > 2 > 1 (d) 2 > 3 > 4 > 1
(d) redox reaction 67. In the redox reaction
63. A compound contains atoms X, Y, Z the oxidation xMnO + yPbO2 + zHNO3 HMnO4
number of X is +2, Y is +5 and Z is –2 The possible + Pb(NO3)2 + H2O
form of the compound is (a) x = 2, y = 5, z = 10
(a) XY1Z2 (b) Y2(XZ3)2 (b) x = 10, y = 2, z = 5
(c) X3(YZ4)2 (d) X3(Y4Z)2 (c) x = 3, y = 7, z = 6
64. When Benzoic acid is titrated with NaOH, varia­ (d) x = 5, y = 2, z = 10
tion of conductance (Y­axis) with addition of NaOH
68. If a current of 80.0 microampere is drawn from a cell
(X­axis) can be shown by
for 100 days, then the number of farads involved are
(a) Y (b) Y
(a) 7.16 ×10–4 (b) 7.16 × 10–3
(c) 14.32 × 10
–3
(d) 3.46 × 10–3
69. The same amount of electricity was passed through
two separate electrolytic cells containing solutions of
X X
(c) Y
nickel nitrate and chromium nitrate respectively. If 0.3
(d) Y
g of nickel was deposited in the first cell, the amount
of chromium deposited is (atomic weight of Ni = 59,
Cr = 52)
(a) 0.130 g (b) 0.236 g
X X (c) 0.176 g (d) 1.76 g
Redox Reactions and Electrochemistry 9.27

70. The charge required to deposit 40.5 g of Al (atomic 77. Cu+(aq) is unstable in solution and undergoes simul­
mass = 27.0 g) from the fused Al2(SO4)3 is taneous oxidation and reduction according to the
(a) 0.434 × 105 C re action,
(b) 4.34 × 105C 2Cu+ (aq) Cu2+ (aq) + Cu (s)
(c) 3.34 × 105C choose correct E° for above reaction if
(d) 43.4 × 105C
E° (Cu2+/ Cu) = 0.34 V and E° (Cu2+/ Cu+) = 0.15 V
71. A certain current liberates 0.504 g of hydrogen in (a) –0.38 V (b) +0.49V
2 hour. The amount (or mass) of copper deposited at
(c) +0.38V (d) –0.19 V
the cathode during the electrolysis of CuSO4 (aq) solu­
tion by the same current flowing for the same time is 78. The standard reduction potentials of Ag, Cu, Co and
(atomic masses H = 1.0, Cu = 63.5) Zn are 0.799, 0.337, –0.277 and –0.762 V respectively.
(a) 16.0 g (b) 32.0 g Which of the following cells will have maximum cell
(c) 8.6 g (d) 1.60 g emf?
(a) Zn | Zn2+ (lM) || Cu2+ (1M) | Cu
72. Which of the following statements are incorrect?
(b) Zn|Zn2+(lM)|| Ag+(lM)|Ag
(1) specific conductance increases with dilution (c) Cu|Cu2+(lM)|| Ag+(lM)|Ag
(2) equivalent conductance decreases with dilution
(d) Zn | Zn2+ (lM) || Co2+ (lM) | Co
(3) the conductance of all electrolytes increase with
temperature 79. On the basis of the information available from the
Select the correct answer using the codes given below reaction, __34 Al + O2 2
__
3 Al2O3,
(a) 1 and 2 (b) 1 and 3
ΔG = –827 kJ mol–1 of O2.
(c) 2 and 3 (d) 1, 2 and 3
The minimum emf, required to carry out an electroly­
73. Specific conductance of 0.01 N solution of an electro­ sis of Al2O3 is (F = 96500 C mol–1)
lyte is 0.00419 mho cm–1. The equivalent conductance (a) 6.42 V (b) 8.56 V
of this solution will be (c) 2.14 V (d) 4.28 V
(a) 4.19 mho cm2
(b) 419 mho cm2 80. One ampere of current is passed for 9650 seconds
(c) 0.0419 mho cm2 through molten AlCl3. What is the weight in grams
of Al deposited at cathode? (Atomic weight of
(d) 0.209 mho cm2
Al = 27)
74. A solution of CuSO4 is electrolyzed for 7 minutes with (a) 0.9 (b) 9.0
a current of 0.6 A. The amount of electricity passed (c) 0.18 (d) 18.0
equal to
81. The emf of a Daniell cell at 298 K is Ei
(a) 26 C (b) 4.2 C
(c) 2.6 × 10–4 F (d) 2.6 × 10–3 F Zn| ZnSO4 || CuSO4 | Cu
(0.01 M) (1.0 M)
75. The specific conductance of 0.1 N KCl solution at
23°C is 0.012 ohm–1 cm–1. The resistance of cell con­ when the concentration of ZnSO4 is 1.0 M and that of
taining the solution at the same temperature was found CuSO4 is 0.01M, the emf changed to E2. What is the
to be 55 ohm. The cell constant will be relationship between E1 and E2?
(a) 0.142 cm–1 (b) 0.616 cm–1 (a) E1 = E2 (b) E2 ≠ E1
(c) 6.16 cm
–1
(d) 616 cm–1 (c) E1 > E2 (d) E1 < E2
76. The equivalent conductance of Ba2+ and Cl– are 82. In the ionic equation
respectively 127 and 76 ohm–1 cm2 equiv–1 at infinite
xBr O–3 + yCr3+ + zH2O Br2 + HCr O–4 + H+
dilution. The equivalent conductance (in ohm–1 cm2
equiv–1) of BaCl2 at infinite dilution will be (a) x = 6, y = 8, z = 10
(a) 139.5 (b) 203 (b) x =10, y = 6, z = 22
(c) 279 (d) 101.5 (c) x = 6, y = 8, z = 10
(d) x = 6, y = 10, z = 22
9.28 Chapter 9

83. The standard electrode potential for electrodes A and B Cu Cu2+ + 2e–; E° = – 0.345 V
are A2+ (aq)/A (s) E° = 0.30 V, B2+ (aq)/B (s) E° = 0.40 V Ag Ag+ + 2e–; E° = – 0.800 V
They are combined in a cell as shown below: Which of the following is most easily reduced?
(a) Fe2+ (b) Ag+
(c) Zn
2+
(d) Cu2+
A(S) B(S)
90. How much chlorine will be liberated on passing one
2+
A aq (1M) 2+
B aq (1M)
ampere current for 30 min through NaCl solution?
(a) 0.66 mol (b) 0.33 mol
Which of the following is the description of such a cell (c) 0.66 g (d) 0.33 g
electrode where positive ions emf/V enter the solution ?
91. Time required to deposit 1 millimol of alumin­
(a) A 0.1 (b) A 0.7
ium metal by the passage of 9.65 ampere of current
(c) B 0.1 (d) B 0.7 through aqueous solution of aluminium ion, is
84. The standard emf of a galvanic cell involving cell (a) 30 s (b) 10 s
reaction with n = 2 is found to be 0.295 V at 25°C. The (c) 30,000 s (d) 10,000 s
equilibrium constant of the reaction would be (Given
92. The chemical reaction,
F= 96500 C mol–1; R = 8.314 JK–1 mol–1)
(a) 2.0 × 1011 (b) 4.0 × 1012 2AgCl (s) + H2(g) 2HCl (aq) + 2Ag (s)
(c) 1.0 × 102 (d) 1.0 × 1010 taking place in a galvanic cell is represented by the
notation
85. A hypothetical electrochemical cell is shown below: (a) Pt (s) | H2 (g), 1 bar | 1 M KCl (aq) |AgCl (s) | Ag
A | A+ (xM) || B+ (yM) | B (s)
The emf measured is +0.20 V. The cell reaction is (b) Pt (s) | H2 (g), 1 bar | 1 M KCl (aq) |1 M Ag+ (aq) |
(a) the cell reaction cannot be predicted Ag(s)
(b) A + B+ A+ + B (c) Pt (s) | H2 (g), 1 bar | 1 M KCl (aq) | AgCl (s) | Ag
(s)
(c) A+ + B A + B+
(d) Pt (s) | H2 (g), 1 bar | 1 M KCl (aq) | Ag (s) | AgCl
(d) A + e
+ –
A; B+ + e– B
93. The emf of the cell
86. If E° (Fe2+/Fe) = –0.441 V and E°(Fe3+/Fe2+) = 0.771 V,
Cu(s)|Cu2+(aq)||Ag+(aq)|Ag(s)
the standard emf of the reaction
E° Cu/Cu2+ = –0.34 V; E° Ag /Ag+ = –0.80 V
Fe + 2Fe3+ 3Fe2+ will be
(a) +0.46 V (b) –0.46 V
(a) 1.212 V (b) 0.111 V
(c) +1.14V (d) –1.14V
(c) 0.330 V (d) 1.653 V
94. One faraday of electricity is passed separately
87. By passing 0.50 ampere current in an aqueous solution
through one litre of one molar aqueous solutions of
0.5 g of an element (Eq. wt = 96.5) is liberated. The
(i) AgNO3 (ii) SnCl4 and (iii) CuSO4. The number
time of passing current in seconds is
of moles of Ag, Sn, and Cu deposited at cathode are
(a) 100 s (b) 500 s respectively
(c) 1000 s (d) 2000 s
(a) 1.0, 0.25, 0.5 (b) 1.0, 0.5, 0.25
88. In the redox reaction (c) 0.5, 1.0, 0.5 (d) 0.25, 0.25, 0.5
xKMnO4 + yNH3 KNO3 + MnO2 + KOH 95. 2MnO–4 + aH+ + bH2O2 2Mn+2 + H2O + dO2.
+ H2O What are the values of a, b, c and d?
(a) x = 3, y = 8 (b) x = 6, y = 3 (a) 4, 5, 4, 5 (b) 6, 5, 8, 5
(c) x = 5, y = 10 (d) x = 8, y = 3 (c) 3, 2, 4, 8 (d) 2, 5, 4, 6
89. The values of standard oxidation potentials of follow­ 96. When X amperes of current is passed through molten
ing reactions are given below: AlCl3 for 96.5 seconds, 0.09 grams of aluminium is
Zn Zn2+ + 2e–; E° = 0.762 V deposited. What is the value of X?
Fe Fe2+ + 2e–; E° = 0.440 V (a) 10 (b) 20
(c) 30 (d) 40
Redox Reactions and Electrochemistry 9.29

97. The electrochemical equivalent of a metal is ‘x’ g. cou­ (a) +1.85 (b) –1.85 V
lomb–1. The equivalent weight of metal is (c) +0.83 V (d) –0.83 V
(a) x (b) x. 96500 105. The reaction potential values of M, N and O are
(c) x/96500 (d) 1.6 × 10–19 +2.46, –1.13 and –3.13 V respectively. Which of the
98. In electrolysis of dilute H2SO4, what is liberated at following order is correct, regarding their reducing
anode? property?
(a) H2 (b) SO 2– (a) O > N > M
4
(b) O > M > N
(c) SO2 (d) O2
(c) M > N > O
99. The hydrogen electrode is dipped in a solution of pH (d) M > O > N
3 at 25°C. The potential would be (the value of 2.303
106. aHNO3 + bH2S cNO + d H2O + 3S. What are
RT/F is 0.059 V)
the values of a, b, c, d?
(a) 0.177 V (b) 0.087 V
(a) 2, 3, 2, 2 (b) 2, 3, 2, 4
(c) 0.059 V (d) –0.177 V.
(c) 3, 3, 2, 4 (d) 4, 3, 1, 2
100. Corrosion of iron is essentially an electrochemical 107. If the aqueous solutions of the following salts are
phenomenon where the cell reactions are electrolysed for 1 hour with 10 ampere current, which
(a) Fe is oxidized to Fe2+ and dissolved oxygen in solution will deposit the maximum mass of the metal
water is reduced to OH– at the cathode? The atomic weights are Fe = 56, Zn =
(b) Fe is oxidized to Fe3+ and H2O is reduced to O2–
2 65, Ag = 108, Hf = 178 and W = 184
(c) Fe is oxidized to Fe and H2O is reduced to O2–
2+
(a) ZnSO4 (b) FeCl3
(d) Fe is oxidized to Fe2+ and H2O is reduced to O2 (c) HfCl4 (d) AgNO3
101. What is the quantity of electricity (in coulombs) 108. Given the standard reduction potentials Zn2+/Zn =
required to deposit all the silver from 250 mL of 1 M –0.74 V, Cl2/Cl– = 1.36 V, H+/½ H2 = 0 V and Fe2+/Fe3+
AgNO3 solution? (Ag = 108) = 0.77 V. The order of increasing strength as reducing
(a) 2412.5 (b) 24125 agent is
(c) 4825.0 (d) 48250 (a) Zn, H2, Fe2+, Cl–
(b) H2, Zn, Fe2+, Cl–
102. The standard reduction potentials of Zn2+ | Zn and Cu2+
| Cu are –0.76 V and +0.34V respectively. What is the (c) Cl–, Fe2+, Zn, H2
cell emf (in V) of the following cell? (RT/F= 0.059) (d) Cl–, Fe2+, H2, Zn
Zn | Zn2+ (0.05 M) || Cu2+ (0.005 M) | Cu 109. In the electrolytic cell, flow of electrons is from
(a) 1.1295 (b) 1.0705 (a) cathode to anode in solution
(c) 1.1 (d) 1.041 (b) cathode to anode through external supply
103. Ksp of BaSO4 is 1 × 10–10. If the ionic conductances (c) cathode to anode through internal supply
of Ba++ and SO2– 4
ions are 64 and 80 ohm–1 cm2 mol–1 (d) anode to cathode through internal supply
respectively, then its specific conductance is
110. In the standardization of Na2S2O3 using K2Cr2O7 by
(a) 1.44 × 10–8 ohm–1 cm–1 iodometry, the equivalent weight of K2Cr2O7 is
(b) 144 × 10–8 ohm–1 cm–1
(a) (molecular weight)/2
(c) 1.44 × 108 ohm–1 cm–1
(d) 144 × 108 ohm–1 cm–1 (b) (molecular weight)/6
(c) (molecular weight)/3
104. Two electrochemical cells (d) same as molecular weight
Zn | Zn2+ || Cu2+ | Cu and Fe | Fe2+ || Cu2+ | Cu are con­ 111. A gas at 1 atm is bubbled through a solution contain­
nected in series. What will be the net emf of the cell at ing a mixture of 1 M Y– and 1 M Z– at 25°C. If the
25°C? reduction potential of Z > Y > X, then
Given: Zn2+ | Zn = –0.73 V, (a) Y will oxidize X and not Z
Cu2+ | Cu = +0.34 V (b) Y will oxidize Z and not X
Fe2+ | Fe = –0.41 V
9.30 Chapter 9

(c) Y will oxidize both X and Z (c) Pt | H2 (g) | HCl (solution) | AgNO3 (solution ) | Ag
(d) Y will reduce both X and Z (d) A
g | AgCl(s) | KCl (solution) | AgNO3 (solution) |
112. The oxidation number of sulphur in S8, S2F2, H2S Ag
respectively, are 119. A solution containing one mole per litre of each
(a) 0, +1 and –2 Cu(NO3)2; AgNO3; Hg2(NO3)2; is being electrolyzed
(b) +2, +1 and –2 by using inert electrodes. The values of standard
(c) 0, +1 and +2 electrode potentials in volts (reduction potentials)
(d) –2, +1 and –2. are
113. A dilute aqueous solution of Na2SO4 is electrolyzed Ag/Ag+ = +0.80, 2Hg / Hg2++ = +0.79
using platinum electrodes. The product at the anode
and cathode are Cu/Cu++ = +0.34, Mg / Mg++ = –2.37
(a) O2, H2 (b) S2O82–, Na
With increasing voltage, the sequence of deposition of
(c) O2, Na (d) S2O82–, H2 metals on the cathode will be
114. The standard reduction potential for Fe2+/Fe and Sn2+/ (a) Cu, Hg, Ag
Sn electrodes are –0.44 and –0.14 volts respectively. (b) Ag, Hg, Cu, Mg
For the cell reaction (c) Mg, Cu, Hg, Ag
Fe2+ + Sn Fe + Sn2+ (d) Ag, Hg, Cu
The standard emf is 120. Electrolysis of dilute aqueous NaCl solution was car­
(a) +0.30 V (b) –0.58 V ried out by passing 10 mili ampere current. The time
(c) +0.58 V (d) –0.300 V required to librate 0.01 mole of H2 gas at the cathode?
115. The standard oxidation potential E° for the half reac­ (a) 9.65 × 104 Sec (b) 19.3 × 104 Sec
tions are as (c) 28.95 × 104 Sec (d) 38.6 × 104 Sec
Zn Zn2+ + 2e–; E° = + 0.76 V 121. The standard reduction potentials at 298 K for the
following half­reactions are given against each
Fe Fe2+ + 2e–; E° = + 0.41 V
Zn2+ (aq) + 2e Zn (s) –0.762
The emf for the cell reaction
Cr3+ (aq) + 2e Cr (s) –0.740
Fe2+ + Zn Zn2+ + Fe is
(a) +1.17 V (b) –0.35 V 2H+ (aq) + 2e H2 (g) 0.000
(c) +0.35 V (d) 0.117 V
Fe3+ (aq) + 2e Fe2+ (aq) 0.770
116. When a lead storage battery is discharged
Which is the strongest reducing agent?
(a) lead is formed
(a) H2(g) (b) Cr(s)
(b) lead sulphate is consumed
(c) SO2 is evolved (c) Zn (s) (d) Fe2+ (aq)
(d) sulphuric acid is consumed 122. For the equation
117. A solution of sodium sulphate in water is electrolyzed
NO3– + 4H+ + e– 2H2O + NO. The number of
using inert electrodes. The products at the cathode and
electrons in its balanced form would be
anode are respectively
(a) 6 (b) 4
(a) O2, H2 (b) O2, Na
(c) 3 (d) 9
(c) O2, SO2 (d) H2, O2
123. Ag | Ag+ (lM) || Ag+(2M) | Ag,
118. The reaction
1 L solution 1 L solution
½ Hg2(g) + AgCl(s) = H+(aq) +Cl–(aq)+ Ag(s) occurs
0.5 F of electricity in the LHS (anode) the 1F electric­
in the galvanic cell
ity in the RHS (cathode) is first passed making them
(a) Pt | H2 (g) | KCl (solution) | AgCl (s) | Ag independent electro cells at 298 K. The emf of the cell
(b) Pt | H2 (g) | HCl (solution) | AgCl (s) | Ag after electrolysis will
Redox Reactions and Electrochemistry 9.31

(a) increase (b) decrease (a) –0.177 V (b) 0.177 V


(c) not change (d) time is also required (c) 1.77 V (d) 0.277 V
124. One litre of 1 M CuSO4 solution is electrolysed. After 131. Three faraday of electricity is passed through aque­
passing 2F of electricity, molarity of CuSO4 solution ous solutions of AgNO3, NiSO4 and CrCl3 kept in
will be three vessels using inert electrodes. The ratio in
(a) M/2 (b) M/4 moles in which the metals Ag, Ni and Cr will be
(c) M (d) 0 deposited is
(a) 1 : 2 : 3 (b) 2 : 3 : 6
125. In acidic medium MnO4– is an oxidizing agent
(c) 6 : 3 : 2 (d) 3 : 2 : 6
MnO4– + 8H+ + 5e– Mn2+ + 4H2O. If H+ ion
concentration is doubled, electrode potential of the 132. When an electric current is passed through acidulated
half cell MnO4–, Mn2+ / Pt will water, 112 mL of hydrogen gas at NTP collects at the
(a) increase by 28.46 mV cathode in 965 seconds. The current passed, in ampere
(b) decrease by 28.46 mV is
(c) increase by 14.23 mV (a) 0.1 (b) 0.5
(d) decrease by 142.30 mV (c) 1.0 (d) 2.0
126. Calculate the weight of copper that will be deposited 133. For the electrochemical cell, M | M+ || X– | X,
at the cathode in the electrolysis of a 0.2 M solu­
E° M+ / M = 0.44 V and E° X/X– = 0.33 V.
tion of copper sulphate, when quantity of electricity,
equal to the required to liberate 2.24 L of hydrogen at From these data, one can deduce that
STP from a 0.1 M aqueous sulphuric acid, is passed (a) M + X M+ + X– is the spontaneous reaction
(Atomic mass of Cu = 63.5)
(b) M+ + X– M + X is spontaneous reaction
(a) 6.35 g (b) 3.17 g
(c) Ecell = 0.77 V
(c) 12.71 g (d) 63.5 g
(d) Ecell = –0.77 V
127. Given that E° (Zn2+/Zn) = –0.763 V and
E° (Cd2+/Cd) = –0.403 V, the emf of the following cell 134. The emf of the following three galvanic cells
Zn | Zn2+ (a = 0.04) || Cd2+ (a = 0.2) | Cd (1) Zn|Zn2+(l M) || Cu2+(1 M) | Cu
is given by (2) Zn | Zn2+ (0.1 M) || Cu2+ (1 M) | Cu
(a) E = +0.36 + [0.059 / 2] [log (0.2/0.004)] (3) Zn | Zn2+ (1 M) || Cu2+ (0.1 M) | Cu
(b) E = – 0.36 + [0.059 / 2] [log (0.2/0.004)] are represented by E1, E2 and E3. Which of the follow­
(c) E = +0.36 + [0.059 / 2] [log (0.004/0.2)] ing statement is true?
(d) E = –0.36 + [0.059 / 2] [log (0.004/0.2)] (a) E2>E1>E3
128. If the pressure of hydrogen gas is increased from 1 arm (b) E3>E2>E1
to 100 atm, keeping the hydrogen ion concentration (c) E1>E2>E3
constant at 1 M, the voltage of the hydrogen half cell (d) E3>E1>E2
at 25°C will be
(a) –0.059 V (b) +0.059 V 135. When a quantity of electricity is passed through CuSO4
solution, 0.16 g of copper gets deposited. If the same
(c) 5.09 V (d) 0.259 V
quantity of electricity is passed through acidulated
129. The conductivity of 0.01 mol/dm3 aqueous acetic acid water, then the volume of H2 liberated at STP will be
at 300 K is 19.5 × 10–5 ohm–1 cm–1 and limiting molar (At. wt of Cu = 64)
conductivity of acetic acid at the same temperature is (a) 4.0 cm3 (b) 56 cm3
390 ohm–1 cm2 mol–1. The degree of dissociation of (c) 604 cm
3
(d) 8.0 cm3
acetic acid is
136. 4.5 g of aluminium (at. mass 27 amu) is deposited at
(a) 0.05 (b) 0.5 × 10–2
cathode from Al3+ solution by a certain quantity of
(c) 5 × 10
–7
(d) 5 × 10–3
electric charge. The volume of hydrogen produced at
130. The hydrogen electrode is dipped in a solution of pH = STP from H+ ions is solution by the same quantity of
3.0 at 25°C. The potential of hydrogen electrode would electric charge will be
be
9.32 Chapter 9

(a) 44.8 L (b) 22.4 L (c) 1 and 3 are correct


(c) 11.2 L (d) 5.6 L (d) 1, 2 and 3 are correct
137. The half cell reaction for the corrosion 142. The reversible reduction potential of pure water is
2H+ + ½ O2 + 2e– H2O, E° = 1.23 V, –0.413 V under 1.00 atm H2 pressure. If the reduction
2+
Fe + 2e –
Fe (s); E° = –0.44 V is considered to be 2H+ + 2e– H2, calculate pH
Find the ΔGO (in kJ) for the overall reaction. of pure water.
(a) –76 (b) –322 (a) 6 (b) 7
(c) –161 (d) –152 (c) 3 (d) 5
138. The emf of the cell 143. For a Ag – Zn button cell, net reaction is
Zn | Zn2+ (0.01 M) || Fe2+ (0.001 M) | Fe at 298 K is Zn(s) + Ag2O (s) ZnO (s) + 2Ag (s)
0.2905 volt. Then the value of equilibrium constant for ΔG°f (Ag2O)= –11.21 kJ mol–1
the cell reaction is ΔG°f (ZnO) = –318.3 kJ mol–1
(a) e0.32/ 0.0295 (b) 100.32/0.0295 Hence E°cell of the button cell is
(c) 100.26/’0.0295 (d) 100.32/0.0591 (a) 3.591 V (b) 2.591 V
139. For the electrolytic production of NaClO4 from NaClO3 (c) –1.591 V (d) 1.591 V
as per the equation,
NaClO3 + H2O NaClO4 + H2. How many fara­ 144. A current of 15 amp is employed to plate Nickel in a
days of electricity will be required to produce 0.5 mole NiSO4 bath. Both Ni and H2 are formed at the cathode.
of NaClO4, assuming 60% efficiency? If 9.9 g of Ni are deposited with the simultaneous lib­
(a) 1.67 F (b) 3.64 F eration of 2.51 litres of H2 measured at STP, what is the
(c) 2.67 F (d) 4.67 F current efficiency for the deposition of Ni? (Atomic
weight of Ni = 58.7)
140. The standard reduction potentials of Cu2+/Cu and Cu2+/
(a) 60 % (b) 70 %
Cu+ are 0.337 V and 0.153 V respectively. The stan­
dard electrode potential of Cu+/Cu half cell is (c) 80 % (d) 56%
(a) 0.184 V (b) 0.827 V 145. Four elements A, B, C and D can form diatomic
(c) 0.521 V (d) 0.490 V molecules and monoatomic anions with –1 charge.
Consider the following reactions about these.
141. Equal quantities of electricity are passed through three
voltameters containing FeSO4, Fe2(SO4)3, and Fe(NO3)3. 2B– + C2 2C– + B2
Consider the following statements in this regard B2 + 2D –
2B– + D2
(1) the amount of iron deposited in FeSO4 and 2A– + C2 no reaction
Fe2(SO4)3 is equal Select correct statement about these.
(2) the amount of iron deposited in Fe(NO3)3 is two (1) A2 is strongest oxidizing agent while D is strong­
thirds of the amount of iron deposited in FeSO4 est reducing agent
(3) the amount of iron deposited in Fe2(SO4)3 Fe(NO3)3 (2) D2 is strongest oxidizing agent while A is strong­
is equal est reducing agent
Of these statements (3) C2 will oxidize B– and also D– to form B2 and D2
(a) 1 and 2 are correct (4) EOA2/A– is the lowest
(b) 2 and 3 are correct (a) 2 and 3 (b) 1 and 3
(c) 2 and 4 (d) 1, 2 and 3

More than One Option Correct Type

146. In which of the following aqueous solutions during 147. For the electrolysis of CuSO4 solution which is/are
electrolysis, H2 and Cl2 are liberated? correct?
(a) CuCl2(aq) (b) KCl(aq) (a) Cathode reaction: Cu2+ + 2e– → Cu using Cu
(c) MgCl2(aq) (d) NaCl(aq) electrode
Redox Reactions and Electrochemistry 9.33

(b) Anode reaction: Cu → Cu2+ + 2e– using Cu (c) KMnO4 is a stronger oxidising agent than K2Cr2O7
electrode in acid medium.
(c) Cathode reaction: 2H+ + 2e– → H2 using Pt electrode (d) K2Cr2O7 oxidises a secondary alcohol to a ketone.
(d) Anode reaction: Cu → Cu2+ + 2e– using Pt
151. When Cl2 is passed through hot NaOH solution, oxida­
electrode
tion number of chlorine changes from
148. Given that (a) 0 to −1 (b) 0 to +5
E0Ni2+ / Ni = −0.25 V; E0Cu 2+ / Cu = +0.34 V (c) 0 to +7 (d) −1 to 0

E0Ag+ / Ag = +0.80 V; E0Zn 2+ / Zn = −0.76 V 152. In which of the following compounds the oxidation
state of oxygen is other than – 2?
Which of the following redox processes will not take (a) H2O2 (b) O2
place in specified direction? (c) O2F2 (d) H2O
(a) Zn(s) + 2H+(aq) → Zn2+ (aq) + H2(g) 153. Identify correct statements for the following incom­
(b) Cu(s) + 2H+ (aq) → Cu2+(aq) + H2(g) plete reactions.
(c) Cu(s) + 2Ag+ (aq) → Cu2+(aq) + 2Ag(s) (i) KI + Cl2 →
(d) Ni2+ (aq) + Cu(s) → Ni(s) + Cu2+ (aq) (ii) KClO3 + I2 →
(iii) KNO3(s) + K(s) →
149. Which is/are correct statement about salt bridge? (iv) P + I2 + H2O →
(a) Ions of salt bridge discharge at electrode (Red)
(b) Ions of salt bridge do not discharge at electrode (a) All are redox reactions
(b) Only I, II and III have one of the product in its
(c) Velocity of ions of salt bridge are almost equal
element form
(d) Salt bridge complete the electric circuit. (c) I and II are displacement reactions
150. Which of the following statements are correct ? (d) III is disproportionation reaction
(a) KMnO4 is a powerful oxidising agent. 154. Identify the compounds in which the sulphur atoms
(b) K
MnO4 is a weaker oxidising agent than K2Cr2O7 are in different oxidation states?
in acid medium. (a) K2S2O7 (b) Na2S2O3
(c) Na2S4O6 (d) K2S2O8

Passage Based Questions

Passage–1 0.059  H Q
(c) E = Eo − log  2 
In an electrolytic cell, electrolysis is carried out. 2  Q 
Electrical energy is converted into chemical energy.
 + 
In an electrochemical cell, chemical reaction, i.e., (d) E = Eo + 0.059 log  [Q][H ] 
redox reaction occurs and electricity is generated. So 2  [H 2Q] 
chemical energy is converted into electrical energy.
Electrolysis is governed by Faraday’s laws. The poten­ 156. The standard reduction potential values of three metal­
tial difference between the electrodes which is called lic cations X, Y and Z are 0.50 V, −3.03 V and −1.2 V
electromotive force is responsible for the generation of respectively. The order of reducing power of the corre­
electric energy in the electrochemical cells. sponding metals is
(a) X > Y > Z
155. The quinhydrone half cell is represented as Pt /H2Q, Q, (b) Z > Y > X
H+ (aq). (c) Y > Z > X
H2Q and Q are present in 1 : 1 molar ratio. The (d) X > Z > Y
electrode potential of the above electrode is written as 157. Two electrolytic cells, one containing acidified FeCl2
(a) E = Eo + 0.059 log[H+] and another acidified FeCl3 are connected in series.
(b) E = Eo − 0.059 log[H+] The ratio of iron deposited at the cathodes in the tow
9.34 Chapter 9

cells will be moving from the top to the bottom in the series, tendency
(a) 3 : 1 to gain electrons, i.e., to get reduced increases. The elec­
(b) 2 : 1 trode systems having negative values of standard reduction
(c) 2 : 3 potentials act as anode when connected to a standard
(d) 3 : 2 hydrogen electrode, while those having positive values act
as cathode.
Passage-2
161. HCl cannot be stored in an aluminium vessel because
For the following cell Pb|PbCl2(s) |PbCl2(solution)|AgCl(s)|Ag
(a) Al is a highly reactive metal.
The potential at 298 K is 0.490 V and the variation of emf (b) HCl is an oxidizing acid
with temperature is given by E = a – (186 × 10–4 VK–1) (c) E0Al3+/Al is much smaller than E0H+/H2
(T – 25 K)
(d) All of these
158. ΔG value is in kJ mol­1
162. If E0cu2+/Cu = 0.34 V and E0Ag+/Ag= 0.8 V, predict whether
(a) – 64.9 (b) 64.9 the reaction given below is feasible or not ?
(c) – 94.6 (d) 94.6
Cu2+ (aq) + 2Ag (s) Cu(s) + 2Ag+ (aq)
159. ΔS value accompanying the cell reaction is on JK–1 (a) Not feasible
mol–1 (b) Feasible
(a) 35.9 (b) – 35.9 (c) Feasible on increasing the conc. of Ag+ (aq).
(c) 49.5 (d) − 38.5 (d) easible at high temp.
160. ΔH value of the cell reaction is in kJ mol–1 163. If E0Fe2+/Fe = −0.44V and E0Mg+2/Mg) = −2.37 V, E0Cu2+/Cu
(a) – 102.57 (b) 52.63 = +0.34 V and E0Ag+/Ag = +0.80 V, the correct order in
(c) – 105.27 (d) 100.5
which the metals displace each other is
Passage-3 (a) Fe > Cu > Ag > Mg
The electrochemical series is the arrangement of various (b) Ag > Cu > Fe > Mg
electrode systems in the increasing order of their standard (c) Mg > Fe > Cu > Ag
reduction potentials. It has several important features. On (d) Fe > Ag > Cu > Mg

Match the Column Type

164. Match the following Column–I Column–II


Column–I (c) 22.4 L of hydrogen at (r) 2 Faraday
(a) Pt |H2 (1 atm)| H+ (1M) STP collected.
(b) Pt | Cl−|Cl2 (d) 8 g of oxygen collected. (t) 5.6 L at STP
(c) Pt|H2(1atm)|H+(C1||H+(C2)H2(1atm) |Pt
(d) Pt|H2(p1atm)|H+(1M)||H+(1M)| H2(p2atm)|Pt
166. Match the following
Column–II
Column–I Column–II
(p) Wrong representation
(q) Concentration cell (a) 50 % solution of H2SO4 (p) H2 is evolved at
(r) Standard hydrogen electrode using Pt electrodes cathode
(s) Eo = 0 (b) Dilute solution NaCl (q) O2 is evolved at
(t) The cell may be commercially feasibly using Pt electrodes anode
165. Match the following (c) Dilute solution of H2SO4 (r) Cl2 is evolved at
using Cu electrodes anode
Column–I Column–II
(d) Concentrated solution of (s) H2S2O8 is
(a) Charge on one mole of (p) 1 Faraday LiCl using Pt electrodes. formed at anode
electron.
(t) non­spontaneous
(b) 108 g of silver deposited (q) 96500 coulomb process
at electrode.
Redox Reactions and Electrochemistry 9.35

Integer Type

167. A solution containing 4.5 mM of Cr2O72− and 15 mM The value n is ______. Given, log 5 = 0.699 and log
of Cr3+ shows a pH of 2.0. The potential of the half 2.575 = 0.4108.
reaction is approximately _____ V.
172. In the electrolysis of KI, I2 is formed at the anode by
168. 6 ampere current was passed through an aqueous the reaction;
solution of Pd salt for 0.5 hour. 2.977 g of Pdn+ was
2I I2 + 2e−
deposited at cathode. The value of n is _____. (Atomic
weight of Pd = 106.4) After the passage of current of 0.5 ampere for 9650
seconds, I2 is formed which required 40 ml of 0.1 M
169. The current strength in ampere required to deposit
Na2S2O3.5H2O solution in the following reaction;
8.0 g of silver in one hour is [At No. of Ag = 108]
approximately _____. I2 + 2S2O32− S4O62− + 2I−
170. The standard electrode potential of Cu2+/Cu = 0.34 V. What is the current efficiency ?
The electrode potential will be zero, when the conc. of
Cu2+ is as x × 10­12 M. the value of x is _____. 173. 500 ml of 0.150 M AgNO3 solution is mixed with 500
ml of 1.09 M Fe2+ solution and the reaction is allowed
[lig 2 = 0.3010, log3 = 0.4771 and log 3.4 = 0.5315] to reach equilibrium at 25oC.

171. A metal ‘S’ is added in excess to 500 ml of 1 Molar Ag+ (aq) + Fe2+ (aq) Fe3+ (aq) + Ag(s)
solution of another metal ion Bn+ at 25oC until the equi­
For 25 ml of the equilibrium solution, 30 ml of 0.0833
librium, S + Bn+ Sn+ + B is reached, If EoSn+/S and Eo
n+ are – 0.75 V and – 0.24 V respectively. Given at
M KMnO4 were required for oxidation. Calculate the
B /B
equilibrium, the concentration of Bn+ is 5.15 × 10­18 M. approximate equilibrium constant for the reaction at
25oC.

Previous Years’ Questions

174. For the following cell with hydrogen electrodes at two EOcell is
different pressures p1 and p2 Pt(H2)|H+(aq)|Pt(H2) (a) x + 2y (b) 2x + y
p1 1M p2 (c) y – x (d) y – 2x
[2002]
177. Consider the following reaction at 1100°C [2002]
emf is given by
(a) RT/F loge p1/p2 (b) RT/2F loge pl/p2 (I) 2C + O2 2CO ΔGO = –460 kJ mol–1
(c) RT//nogep2/p, (d) RT/2Flogep2/Pl
(II) 2Zn + O2 2ZnO ΔGO = –360 kJ mol–1
175. Which of the following reaction is possible at an ode? based on these, select correct alternate
[2002] (a) zinc can be oxidized by CO
(a) F2 + 2e –
2F – (b) zinc oxide can be reduced by carbon
(b) 2H+ + ½ O2 + 2e– H2O (c) both are correct
(c) 2Cr2 + 7H2O
3+
Cr2O72– + 14 H+ + 6e– (d) none is correct
(d) Fe 2+ 3+
Fe + e –
178. Conductivity (Seimens S) is directly proportional to
area of the vessel and the concentration of the solution
176. For a cell given below [2002]
in it and is inversely proportional to the length of the
Ag | Ag+ || Cu2+ iCu vessel, then constant of proportionality is expressed in
– + [2002]
Ag+ + e– Ag, E° = x (a) S m mol–1 (b) S2 m2 mol–2
(c) S m2 mol–1 (d) S2 m2 mol
Cu2+ + 2e– Cu, E° = y
9.36 Chapter 9

179. For the redox reaction 186. The limiting molar conductivities ΛO for NaCl, KBr
and KCl are 126, 152 and 150 S cm2 mol–1 respectively.
Zn (s) + Cu2+ (0.1 M) Zn2+ (1M) + Cu (s) The ΛO for NaBr is [2004]
Taking place in a cell, E°cell is 1.10 volt. Ecell for the cell (a) 278 S cm mol
2 –1

will be (2.303 RT/F = 0.0591) [2003] (b) 178 S cm2 mol–1


(a) 2.14 V (b) 1.80 V (c) 128 S cm2 mol–1
(c) 1.07 V (d) 0.82 V (d) 306 S cm2 mol–1
180. For a cell reaction involving two electrons, the standard 187. In a cell that utilizes the reaction [2004]
emf of the cell is found to be 0.295V at 25°C. The equi­ Zn (s) + 2H+ (aq) Zn2+ (aq) + H2 (g)
librium constant of the reaction at 25°C will be [2003]
Addition of H2SO4 to cathode compartment, will
(a) l × 10–10 (b) 29.5 × 10–2
(c) 10 (d) 1 × 1010 (a) lower the E and shift equilibrium to the right
(b) lower the E and shift the equilibrium to the left
181. During electrolysis of a solution of AgNO3, 9650 cou­ (c) increase the E and shift the equilibrium to the left
lombs of charge pass through the electroplating bath, the (d) increase the E and shift the equilibrium to the
mass of silver deposited on the cathode will be [2003] right.
(a) 1.08 g (b) 10.8 g
(c) 21.6 g (d) 108 g 188. The E (M3+/M2+) values for Cr, Mn, Fe and Co are
–0.41, +1.57, +0.77 and +1.97 V respectively. For
182. Standard reduction electrode potentials of three metals which one of these metals the change in oxidation state
A, B and C are +0.5 V, –3.0 V and –1.2 V respectively. form +2 to +3 is easiest? [2004]
The reducing power of these metals are [2003] (a) Cr (b) Mn
(a) B > C > A (b) A > B > C (c) Fe (d) Co
(c) C > B > A (d) A > C > B
189. For a spontaneous reaction the ΔG, equilibrium con­
183. In a hydrogen­oxygen fuel cell, combustion of hy dro­ stant (K) and E°cell will be respectively [2005]
gen occurs to [2004] (a) –ve, > 1, +ve
(a) produce high purity water (b) +ve, > 1, –ve
(b) generate heat (c) –ve, < 1, –ve
(c) remove adsorbed oxygen from electrode surfaces (d) –ve, > 1, –ve
(d) create potential difference between the two
electrodes 190. Aluminium oxide may be electrolysed at 1000°C to fur­
nish aluminium metal (atomic mass = 27 amu; 1 fara­
184. Consider the following E° values [2004] day = 965000 coulombs). The cathode reaction is
E (Fe3+/Fe2+) = + 0.77 V Al3+ + 3e– Al
To prepare 5.12 kg of aluminium metal by this method
E(Sn2+/Sn) = –0.14 V
would require [2008]
Under standard conditions, the potential for the reaction (a) 5.49 × 107 C of electricity
(b) 1.83× 107 C of electricity
Sn(s) + 2Fe3+ (aq) 2Fe2+ (aq) + Sn2+ (aq) is
(c) 5.49 × 104 C of electricity
(a) 1.68 V (b) 0.91 V
(d) 5.49 × 1010 C of electricity
(c) 0.63 V (d) 1.46 V
191. Electrolyte KCl KNO3 HCl NaOAc NaCl
185. The standard emf of a cell, involving one electron
(Scm2 149.9 145 426.2 91.0 126.5 mol–1)
change is found to be 0.591 V at 25°C. The equilib­
rium constant of the reaction is (F = 96500 C mol–1, R Calculate using appropriate molar conductances of the
= 8.314 JK–1 mol–1) [2004] electrolytes listed above at infinite dilution in H2O at
(a) 1.0 × 1030 (b) l.0 × 101 25°C [2008]
(c) 1.0 × 105 (d) 1.0 × 1010 (a) 517.2 (b) 552.7
(c) 390.7 (d) 217.5
Redox Reactions and Electrochemistry 9.37

192. The electrical conductivity of the flowing aqueous 198. In a fuel cell methanol is used as fuel and oxygen gas
solutions is highest for [2008] is used as an oxidizer. The reaction is
(a) 0.1 M CH3COOH (b) 0.1 M CH2FCOOH CH3OH (l) + 3/2 O2 (g) CO2 (g) + 2H2O(l)
(c) 0.1 M CHF2COOH (d) 0.1 M CH2ClCOOH
At 298 K standard Gibb’s energies of formation for
193. Given the data at 25°C [2006] CH3OH (l), H2O (l) and CO2 (g) are –166.2. –237.2
Ag + I– AgI + e–; E° = 0.152 V and –394.4 kJmol–1 respectively. If standard enthalpy
Ag Ag + e– ; E° = –0.800 V
+
of combustion of methanol is –7269 kJ mol–1, effi­
ciency of the fuel cell will be: [2009]
What is the value of log Ksp for AgI?
(a) 87% (b) 90%
(2.303 RT/F = 0.059 V) (c) 97% (d) 80%
(a) –8.12 (b) +8.612 199 The correct order of E0M2+ / M values with negative sign
(c) –37.83 (d) –16.13 for the four successive elements Cr, Mn, Fe and Co is
194 Resistance of a conductivity cell filled with a solution [2010]
of an electrolyte of concentration 0.1 M is 100 Ω. The (a) Mn > Cr > Fe > Co (b) Cr > Fe > Mn > Co
conductivity of this solution is 1.29 S m–1. Resistance (c) Fe > Mn > Cr > Co (d) Cr > Mn > Fe > Co
of the same cell when filled with 0.2 M of the same
200. The Gibbs energy for the decomposition of Al2O3 at
solution is 520 Ω. The molar conductivity of 0.02 M
500°C as follows [2010]
solution of the electrolyte will be [2006]
(a) 124 × 10 S m mol
–4 2 –1
(b) 1240 × 10 S m2 mol–1
–4
2 4
Al 2 O3 Al + O 2
(c) 1.24 × 10 S m mol
–4 2 –1
(d) 12.4× 10–4 S m2 mol–1 3 3
195. The equivalent conductances of two strong electro­ ΔGR = 966 kJ/mole
lytes at infinite dilution in H2O (where ions move The potential difference needed for electrolytic reduc­
freely through a solution) at 25°C are given below: tion of Al2O3 at 500°C is at least
[2007] (a) 4.5 V (b) 2.5 V
Λ° (CH3COONa) = 91.0 S cm /equiv. 2
(c) 3 V (d) 5
Λ° (HCl) = 426.2 S cm2/equiv.
201. The reduction potential of hydrogen half­cell will be
What additional information/quantity one needs to negative if: [2011]
calculate Λ° of an aqueous solution of acetic acid? (a) p(H2) = 2 atm and [H+] = 1.0 M
(a) Λ° of CH3COOK (b) p(H2) = 1 atm and [H+] = 1.0 M
(b) The limiting equivalent conductance of H+ (λ°) (c) p(H2) = 1 atm and [H+] = 2.0 M
(c) Λ° of chloroacetic acid (ClCH2COOH) (d) p(H2) = 2 atm and [H+] = 2.0 M
(d) Λ° of NaCl
202. The standard reduction potentials for Zn2+/Zn, Ni2+/Ni,
196. The cell Zn | Zn2+ (1 M) || Cu2+ (1 M) | Cu E°cell = 1.10 and Fe2+/Fe are –0.76, –0.23 and –0.44 V respectively.
V), was allowed to be completely discharged at 298 The reaction X + Y2+ → X2+ + Y will be spontaneous
K. The relative concentration of Zn2+ and Cu2+ ([Zn2+]/ when [2012]
[Cu2+]) is [2007]
(a) X = Fe, Y = Zn (b) X = Ni, Y = Zn
(a) 37.3 (c) X = Ni, Y = Fe (d) X = Zn, Y = Ni
(b) 1037.3
(c) 9.65 × 104 203. Given [2013]
(d) antilog (24.08) 0
ECr3+ / Cr = −0.7; E0MnO− / Mn2+ = 1.51V
197. Given E° Cr / Cr = –0.72 V, E°Fe / Fe = –0.42 V. The
3+ 2+ 4

0
potential for the cell [2008] 0
ECr O2− / Cr3+ = 1.33V; ECl / Cl = 1.36 V
2 7

Cr | Cr3+ (0. 1 M) || Fe2+ (0.01 M) | Fe is Based on the data given above, strongest oxidisting
(a) 0.26 V (b) 0.399 V agent will be
(c) –0.339 V (d) –0.26 V (a) Mn2+ (b) MnO4

(c) Cl– (d) Cr3+


9.38 Chapter 9

204. Resistance of 0.2 M solution of an electrolyte is 50 Ω 206. Given below are the half­cell reactions: [2014]
The specific conductance of the solution is 1.4 S m–1.
The resistance of 0.5 M solution of the same electro­ Mn2+ + 2e– → Mn; Eo = −1.18 V
lyte is 280 Ω The molar conductivity of 0.5 M solu­ 2(Mn3+ + e– → Mn2+); Eo = +1.51 V
tion of the electrolyte in S mt2 mol–1 is: [2014]
(a) 5 × 10
3
(b) 5 × 102 The Eo for 3Mn2+ → Mn + 2Mn3+ will be:
(c) 5 × 10 –4 (d) 5 × 10 –3 (a) –0.33 V; the reaction will not occur
(b) –0.33 V; the reaction will occur
205. The equivalent conductance of NaCl at concentration
(c) –2.69 V; the reaction will not occur
C and at infinite dilution are λC and λ∞, respectively.
(d) –2.69 V; the reaction will occur
The correct relationship between λC and λ∞ is given as:
(where the constant B is positive) [2014] 207. Two faraday of electricity is passed through a solution
(a) λC = λ∞– (B) C of CuSO4. The mass of copper deposited at the cathode
(b) λC = λ∞+ (B) C is (at. mass of Cu = 63.5 amu) [2015]
(c) λC = λ∞ + (B) C (a) 0 g (b) 63.5 g
(c) 2 g (d) 127 g
(d) λC = λ∞ – (B)C

ANSWER KEYS

Single Option Correct Type


1. (d) 2. (a) 3. (c) 4. (b) 5. (a) 6. (c) 7. (b) 8. (b) 9. (d) 10. (d)
11. (c) 12. (d) 13. (c) 14. (d) 15. (c) 16. (a) 17. (c) 18. (a) 19. (c) 20. (d)
21. (b) 22. (a) 23. (d) 24. (b) 25. (b) 26. (c) 27. (c) 28. (b) 29. (a) 30. (b)
31. (d) 32. (b) 33. (d) 34. (a) 35. (b) 36. (c) 37. (b) 38. (a) 39. (c) 40. (a)
41. (b) 42. (a) 43. (c) 44. (a) 45. (b) 46. (b) 47. (d) 48. (d) 49. (c) 50. (c)
51. (c) 52. (b) 53. (a) 54. (d) 55. (a) 56. (b) 57. (d) 58. (c) 59. (b) 60. (a)
61. (a) 62. (c) 63. (c) 64. (b) 65. (a) 66. (d) 67. (a) 68. (b) 69. (c) 70. (b)
71. (a) 72. (a) 73. (b) 74. (d) 75. (b) 76. (a) 77. (c) 78. (b) 79. (c) 80. (a)
81. (c) 82. (d) 83. (d) 84. (d) 85. (b) 86. (a) 87. (c) 88. (d) 89. (b) 90. (c)
91. (a) 92. (b) 93. (a) 94. (a) 95. (b) 96. (a) 97. (b) 98. (d) 99. (d) 100. (a)
101. (b) 102. (b) 103. (b) 104. (a) 105. (a) 106. (b) 107. (d) 108. (d) 109. (c) 110. (b)
111. (a) 112. (a) 113. (a) 114. (d) 115. (c) 116. (d) 117. (d) 118. (b) 119. (d) 120. (b)
121. (c) 122. (c) 123. (c) 124. (d) 125. (a) 126. (a) 127. (c) 128. (a) 129. (a) 130. (a)
131. (c) 132. (c) 133. (b) 134. (a) 135. (b) 136. (d) 137. (b) 138. (b) 139. (a) 140. (c)
141. (b) 142. (b) 143. (d) 144. (a) 145. (b)

More than One Option Correct Type


146. (b), (c) and (d) 147. (a) and (b) 148. (b), and (d) 149. (b), (c) and (d)
150. (a), (c) and (d) 151. (a) and (b) 152. (a) (b) and (c) 153. (a) (b) and (c)
154. (b) and (c)
Redox Reactions and Electrochemistry 9.39

Passage Based Questions


155. (a) 156. (c) 157. (d) 158. (c) 159. (b) 160. (c) 161. (d) 162. (a) 163. (c)

Match the Column Type


164. (a) ­ (r, s); p (b) ­ (p); (c) ­ (q), (s) and (t); (d) ­ (q), (s) and (t)
165. (a) ­ (p, q); (b) ­ (p), (q); (c) ­ (r) and (d) (p) (q), (t)
166. (a) ­ (p), (q), (s), (t); (b) ­ (p), (q), (t); (c) ­ (p), (t); (d) ­ (p), (r),(t)

Integer Type
167. (1) 168. (4) 169. (2) 170. (3) 171. (2) 172. (8) 173. (3)

Previous Year’s Questions


174. (b) 175. (d) 176. (c) 177. (b) 178. (c) 179. (c) 180. (d) 181. (b) 182. (a) 183. (d)
184. (b) 185. (d) 186. (c) 187. (c) 188. (a) 189. (a) 190. (a) 191. (c) 192. (c) 193. (d)
194. (a) 195. (d) 196. (b) 197. (a) 198. (c) 199. (a) 200. (b) 201. (a) 202. (d) 203. (b)
204. (c) 205. (a) 206. (c) 207. (b)
9.40 Chapter 9

HINTS AND SOLUTIONS

Single Option Correct Type


6. Fe2+/Fe E° = –0.44 At cathod
Fe3+/Fe2+ E° = 0.77 2H2O + 2 e– H2 + 2 OH–
The metals having higher negative electrode potential can H2O H2 + ½ O2
displaced metals having lower values of negative electron 31. The weight of deposit = Z. I. t
potential from their salt solutions. Hence Temperature has no role here.
7. 2KBr + I2 2KI + Br2 32. 2H2O 2H2 + O2
reaction is not possible because Br– ion is not oxidized in Br2 2 vol 1 vol
with I2 due to higher electrode potential of I2 than bromine. 5.6 cm3 2.8 cm3
8. In electrolysis of NaCl when Pt electrode is taken, then H2 is 44. During charging of an acid cell, the external source supplies
liberated at cathode, while with Hg cathode it forms sodium current due to which its emf increases and also the relative
amalgam, because more voltage is required to reduce H+ at density of cell increases.
Hg than Pt. 47. As reduction occurs at Ag electrode, an increase in the con­
9. In the silver plating of copper, K[Ag(CN)2] is used instead of centration of Ag+ or decreases in concentration of Cu2+ will
AgNO3 because of less availability of Ag+ ions, as Cu cannot increases the voltage.
displace Ag from [Ag(CN)2–] ion. 48. When a cell reaction takes place, electrical energy is pro­
11. O2PtF6 can be written as O2 + [PtF6]–. In this compound oxida­ duced. The electrical work thus done by the system (cell)
tion state of Pt is +6 and O2 carries a charge of +1. results in the corresponding decreases in the free energy of
Let X = oxidation state of oxygen in O+2 the system.
So, 2X = +1 50. Ionic mobilities (or transport numbers) of K+, NO–3 are nearly
X = + ½ same.
12. ATP has phosphate group, P O3– 51. This is an example of a disproportionation reaction.
4
So, X + 4(–2) = –3 3[Cl O]–1 ClO–3 + 2Cl–1
X = +5. 52. Since the ionic character decreases in the order KCl >
13. I2 + 2 e– 2I– NaCl > LiCl
Molar mass or molecular weight
__________________________________ 53. Lower the reduction potential, stronger is the reducing agent.
Eq. wt = Number of electrons involved in the half­reaction
58. Mg and Al have lower reduction potential than water. So
= ½ the molecular weight of I2. water is reduced more easily to give H2 gas at the cathode.
16. As E = M/change in oxidation number 59. Copper is more reactive than Ag, so it displaces silver from
= M/1 =M AgNO3.
18. The balanced equation is Cu + 2 Ag+ Cu2+ + 2Ag
2MnO–4 + 5C2O2– + 16H+ 60. Charge of one mole of electrons = 96500 C.
4
2Mn + 10CO2 + 8H2O
2+ 61. Equilibrium is attained when Q = Kc
24. As the possible compound must have zero sum of oxidation At equilibrium the cell will not do any electrical work
numbers (ΔG = nFE = 0) and hence, E = 0.
AB2C2 = +6 + 2 × (–2) + 2 × (–1) = 0 0 = E°–(RT/nF) ln Q
27. Here oxidation number of chlorine atom changes from = E°–(RT/nF) ln K
0 to +5. 63. As the possible compound must have zero sum of oxidation
3Cl2 + 6NaOH NaClO3 + 5NaCl + 3H2O numbers
M
________________________ that is, X3(YZ4)2 =3 × 2 + 2 × 5 + 8 × (–2) = 0
28. E = Total change in oxidation number
67. The balanced equation is
or
2MnO + 5PbO2 + 10HNO3 2HMnO4 +
Total number of electrons involved in the reaction
E = M/6 5Pb(NO3)2 + 4H2O
It
_____
30. At anode 68. Number of faraday = 96500
H2O 2H+ + ½ O2 + 2 e– 80 × 10–6 × 100 × 24 × 60 × 60
______________________
= 96500
= 7.16 × 10–3
Redox Reactions and Electrochemistry 9.41

MNi
___ Eq. mass of Ni
___________ 2 × 0.295
_______
69. M = Eq. mass of Cr log10 K = 0.0591 or K = 1010
Cr

0.3g ____
____ 59/2 85. Ecell = +ve, thus redox changes are
MCr = 52/3 A A+ + e– LHS shows oxidation
0.3g × (52/3)
__________
Mcr = (59/2) = 0.176g B+ + e B RHS shows reduction
70. Al3+ + 3e– Al A + B +
A+ + B
= 3 × 96500 C 3F 1 mol = 27.0 g 86. E°(Fe /Fe) = –0.441 V
2+

The required charge E°(Fe3+/Fe2+) = 0.771 V


If E°cell = E°op (Fe/Fe2+) + E°RP (Fe3+/Fe2+)
40.5 g × 3 mol × 96500 C mol–1
______________________
= = 4.34 × 105 C = + 0.441 + 0.771 = 1.212 V
27.0 g
92. 2AgCl (s) + H2 (g) 2HCl (aq) + 2Ag (s)
Eq. mass of hydrogen
Mass of hydrogen __________________
_______________
71. = the activities of solids and liquids are taken as unity and at
Mass of copper Eq. mass of copper
low concentrations, the activity of a solute is approximate by
0.504g ______
_____ 1 equal to its molarity.
MCu = (63.5/2)
The cell reaction will be
0.504g × 63.5
__________ Pt(s) | H2 (g), 1 bar | H+ (aq) 1 M | AgCl (aq) 1 M | Ag(s)
MCu = 2
= 16.0g
95. The balanced equation is
72. Specific conductance decreases with dilution, where as the 2MnO–1 + 6H+ + 5H2O2
equivalent conductance increases with dilution and conduc­
tance of all electrolytes increase with tem perature as the ions 2Mn+2 + 8H2O + 5O2
move faster at higher tempera ture. So a, b, c and d are 6, 5, 8 and 5 respectively.
0.004 mho cm–1
______________ 96. At cathode
73. K = 2H+ + 2e– H2
0.01 eq L–1
0.00419 × 1000 mho cm2 eq–1
_____________________ 4H+ + 4e– 2H2
Λeq = K/Ceq = At anode
0.01

(1 L = 1000 cm2) 2H2O O2 + 4H+ + 4e–


So, O2 is liberated at anode.
= 419 mho cm2 eq–1. 99. pH = 3, [H+] = 10–3
74. The amount of electricity passed = t Ered = Ered
°
+ 0.059 log [ion]
= 0.6 A × 7 × 60 s = 0.6 × 7 × 60 As E = 0 + 0.059 log [10–3]
1F
______ E = + 0.059 (–3) = –0.177 V.
= 252 C × = 2.6 × 10–3 F
96500
100. Fe Fe2+ + 2e– (anode reaction)
76. The equivalent conductance of BaCl2 at infinite dilu tion. O2 + 2H2O + 4e– 4OH– (cathode reaction)
λ∝ of BaCl2 = ½ λ∝ of Ba++ + λ∝ of Cl– The overall reaction is
127 + 76
_______ 2Fe + O2 + 2H2O 2Fe(OH)2.
= = 139.5 ohm–1 cm2
2 Fe(OH)2 may be dehydrated to iron oxide FeO, or further
79. ΔG = –nEF oxidized to Fe(OH)3 and then dehydrated to iron rust, Fe2O3.
For 1 mol of Al, n = 3 106. The balanced equation is
4
__ 4
__
For 3 mol of Al, n = 3 × 3 = 4 2HNO3 + 3 H2S 2NO + 4H2O + 3S
So, 827 × 1000 = 4 × E × 96500 So a, b, c and d are 2, 3, 2 and 4 respectively.
or E = 2.14 V 109. In an electrolytic cell, electrons do notflow them­
selves. It is the migration of ions towards oppositely
81. Cell concentration is Zn + Cu2+ Zn2+ + Cu charged electrodes that indirectly constitutes the flow
RT
___ [Zn2+]
_____ of elect rons from cathode to anode through internal
Ecell = E°cell – In 2+ supply.
nF [Cu ]
110. Cr2O–2 + 14H+ + 6e– 2Cr3+ + 7H2O
greater the factor [(Zn2+) / (Cu2+)], less is the emf. So E1 > E2. 7
Since K2Cr2O7 accepts 6 electrons for its reduction to Cr3+
0.0591
______
84. E° = n log10 K ions.
Molecular weight
_____________
Eq. wt. =
Here, n = 2, E° = 0.295 6
9.42 Chapter 9

111. Higher the reduction potential, better the oxidizing agent. 120. i = 10 × 10–3 = 0.01 anp
Since the reduction potential of Y is greater than X but less q = 0.01 × 2 F
than Z, hence. Y will oxidize X, but not Z. q 0.01 × 2 × 96500
112. S8 elemental state t= =
i 0.01
S2 F2 = 19.3 × 104 sec
2x – 2 = 0 121. More negative the reduction potential, higher will be the
2x = 2. So, x = 1 reducing property, that is, the tendency to give up electrons.
H2 S 122. NO3– + 4H+ + 3e– 2H2O + NO
x + 2 = 0 [Ag+]R
0.0591
_____ ______
x = –2 123. E = E° + 1 log [Ag+]
L

113. During the electrolysis of aqueous Na2SO4, H2 is evolved at = O + 0.0591 log 2 = 0.0591 × 0.301 V
the cathode as ERP of sodium (–2.71 V), is much lower than after current is passed [Ag+]R = 1M
that of water (–0.83 V), while O2 is evolved at the anode as
[Ag+]L = 0.5 M
EaRP potential of SO2–
4
ion (–0.2 V) is much less than that of
water (–1.23 V). Hence, there is change in emf.
124. 1 L of 1 M CuSO4 = 1 mol of Cu2+
114. E°cell = E°c – E°a Cu2+ + 2e– Cu
= –0.44 – (–0.14) = –0.44 + 0.14 = –0.30 V 2F of electricity will convert dilute Cu2+ into Cu, hence
115. The given values are oxidation potential. Change these to [CuSO4] is 0
reduction potential values 0.0591
_____
5 log (2)
8
125. E = E° +
emf = E°c – E°a 0.0591
_____
= E° [Fe2+/Fe] – E° [Zn2+/Zn] = –0.41 – (– 0.76) E – E° = 5 × 8 log 2
= –0.41 + 0.76 = 0.35 V = 0.02846 V = 28.46 mV
116. Reactions during discharging of lead storage battery are 126. At STP mass of 22.4 L H2 = 2 g
At anode Mass of 2.24 L H2 at STP = 0.2 g
Pb(s) + SO2– (aq) PbSO4 (s) + 2e– Mass of copper deposited
__________________
4
Mass of hydrogen
Eq. mass of copper
________________
At cathode = Eq. mass of hydrogen
PbO2 (s) + SO2– 4
(aq) + 4H+ (aq) + 2e– MCu _____
____ 63.5/2
PbSO4 (s) + 2H2O = 1
0.2g
Net reaction
63.5 g × 0.2
_________
Pb(s) + PbO2 (s) + 4H+ (aq) + 2S O 2– 4
(aq) MCu = 2
= 6.35 g.
2PbSO4 (s) + 2H2O
127. Since E° (Cd2+/Cd) > E°(Zn2+/Zn), therefore Zn electrode
Thus H2SO4 is consumed in this reaction.
acts as anode and Cd electrode as cathode.
117. Water is reduced at the cathode and oxidized at the anode
At anode
instead of Na+ and SO2– .
4
Zn (s) Zn2+ (aq.) + 2e–
At cathode
At cathode
2H2O + 2e– H2+ 2OH–
Cd2+ + 2e– Cd (s)
At anode
Zn (s) + Cd2+ (aq) Zn2+ (aq) + Cd (s)
H2O 2H+ + ½ O2 + 2e–
Q = [Zn2+]/[Cd2+] = 0.004/0.2
118. At anode
H2 2H+ + 2e– E_cell^° = E_cathode^° – E_anode^°
At cathode = –0.403 V – (– 0.763 V) = 0.36 V
0.059
_____ 0.004
_____
AgCl + e– Ag + Cl– Ecell = 0.36 V – 2 V log10 0.2
119. The reduction potentials (as given) of the ions are in the
128. 2H+ (1 M) + 2 e– H2 (g, 100 atm)
order
[H (g)]2 ____
______ 100
Ag+ > Hg22+ > Cu2+ > Mg2+ Q = [H+]2 = (1)2 = 100
Mg2+ (aq) will not be reduced, as its reduction potential is 0.059
_____
E(H+/H2) = E(H+ / H2) – 2 V log10 Q (at 25°C)
much lower than water (–0.83 V)
0.059
_____
Hence the sequence of deposition of the metals will be Ag, = 0.0 V – 2 V log10 100 = –0.059 V
Hg, Cu.
Redox Reactions and Electrochemistry 9.43

K
________ 2.303 RT
_______ [Zn ]
_____
2+
129. Λmc = Molar conc 134. Ecell = E°cell – log10 [Cu2+]
nF
19.5 × 10–5 × ohm–1 cm–1
_________________ 2.303 RT
= 0.01 mol L–1 E1 = E°cell –
_______ 1
__
log10 1 = E°cell
2F
19.5 × 10 ohm cm × 1000 cm
–5 –1
________________________ –1 3
2.303 RT
= 0.01 mol E2 = E°cell –
_______ 0.1
___
log10 1.0
2F
= 19.5 ohm–1 cm2 mol–1 2.303 RT
_______ 1
___
cm E3 = E°cell – 2F
log10 0.1
___
Degree of dissciation (α) = Λ

m Wt of Cu deposited
______________ Eq. wt of Cu
_________ 64/2
____
19.5 ohm–1 cm2 mol–1
_______________ 135. Wt of H produced = Eq. wt of H = 1
= 390 ohm–1 cm2 mol–1 = 0.05 2

0.16
_______ 32 _____
__ 22400
130. pH = 3.0 or [H+] = 10–3 M Wt of H2
= 1 = 2 × 5 × 10–3 cc = 56 cc
2H+ + 2e– H2 (g)
136. 1 faraday charge liberates 1 equivalent of substance.
+ 0.059
_____ + 1
____
E(H /H2) = E°(H /H2) – 2 log10 [H+]2 Eq. wt of Al = 27/3 = 9
Wt of Al
_______ 4.5
___
Number of equivalent of Al = Eq. wt = 9 = 0.5
0.059
_____ 1
_____
= 0.0 V – 2 log10 (10–3)2
Number of faraday required = 0.5
0.059
_____
= 0.0V – 2 log10 (103)2 Number of equivalent of H2 produce = 0.5 equivalent
–0.059
_____ 22.4
____
= 2 × 6 = –0.177 V Volume occupied by 1 equivalent of H2 = 2 = 11.2

131. (i) Ag+ (aq) + e– Ag (s) Volume occupied by 0.5 = 11.2 × 0.5
1 mol = 1F 1 mol = 5.6 litre of STP
3F 3 mol 137. Fe (s) Fe2+ + 2e–; ΔGO1.
(ii) Ni2+ (aq) + 2e– Ni (s) 2H + 2e + ½ O2
+ –
H2O (I)–; ΔG°2
2 mol = 2 F 1 mol
Fe(s) + 2H + ½ O2
+
Fe2+ + H2O ΔG°3
3F 3/2 mol
ΔG°2 + ΔGO1 = ΔG°3
(iii) Cr3+ (aq) + 3e– Cr (s)
3 mol = 3F 1 mol ΔG°3 = (– 2F × 0.44) + (– 2F × 1.23)
The required ratio of moles of Ag, Ni and Cr is 3 mol = – (2 × 96500 × 0.44 + 2 × 96500 × 1.23)
Ag : 3/2 mol Ni: 1 mol Cr = –32 2310 J = –322 kJ.
that is 6 mol Ag : 3 mol Ni: 2 mol Cr (6 : 3 : 2) 138. Zn + Fe2+ Zn2+ + Fe (n = 2)
132. 22400 mL of hydrogen at STP (or NTP) = 2 g 0.0591
_____
E = E° – n log Kc
So, 112 mL of hydrogen at STP
0.0591
_____ 0.01
_____
2 g × 112 mL 0.0295 = E° – log 0.001
= __________ = 10–2 g 2
22400 mL
E° = 0.2905 + 0.0295 = 0.32 volt
2H + 2e–
+
H2 0.0591
_____
2F 1 mol E° = n log Keq
0.0591
_____
= 2 × 96500 C = 2g 0.32 = 2 log Keq = 0.02945 log Keq
2 g hydrogen is deposited by = 2 × 96500 C Keq = 100.32/0.0295
10–2 g hydrogenwill be deposited by
139. Oxidation reaction is
2 × 96500 ×10–2 g
_______________
= = 965 C H2O + Cl O –3 Cl O –4 + 2e– + 2H+
2g
Charge (Q) = T Oxidation number of CI changes from +5 to +7. So, for pro­
Q _____
__ 965 C duction of 1 mole of perchlorate, 2F are required.
= T = 965 s = 1 Cs–1 = 1A
For 0.5 mole, 1 F required
133. M+ + e– M; Eº = + 0.44 V
Since efficiency is 60% only,
X + e– X–; Eº = + 0.33 V 1.0/0.6 = 1.67 F are required
Now in case of (b)
140. Cu2+ + 2e– Cu; (i)
°
Emf = E°c – E°a
as ΔGO1 = –nFE° = –2F × 0.337
= E° (M+/M) – E° (X/X–)
For the process (ii)
= 0.44 – 0.33 = 0.11 V Cu2+ + e– Cu+;
Thus, M+ + X– M + X is a spontaneous reaction. ΔG°2 = –1F × 0.153
9.44 Chapter 9

Now subtract equation (ii) from (i) 142. 2H+ + e– H2


Cu+ + e– Cu; ΔG°3 = ΔGO1 – ΔG°2 Q = 1/[H+]2
= –0.674 F – (–0.153 F) 0.0591
_____ 1
_____
E = E°SHE – 2
log [H+]2
= –0.674 F+ 0.153 F
0.0591
_____
= 0.521 F –0.413 = 0 + 2 log [H+]2–
Now, –nFE°Cu2+/Cu = –0.521 F 0.413 = –0.0591 log [H+]
E°Cu2+/Cu= –0.521 V. (as – log [H+] = pH)
141. Electrode reaction for = 0.0591 pH
FeSO4 Fe2+ + 2e– = Fe; n = 2 0.413
_____
Fe2(SO4)3 Fe3+ + 3e– = Fe; n = 3 pH = 0.0591 = 7
Fe(NO3)3 Fe3+ + 3e– = Fe; n = 3 143. ΔG° = ΔGf(ZnO) – ΔGf(Ag2O)
Mass of Fe deposited (m) by the current (I) passed for ‘t’ = –318.30 + 11.21 = –307.09 kJ
seconds is given by = –307.09 × 103 J
M
__
m = nF I.t or m ∝ n
M
__ ΔG = –nFEcell
O °

–307.09 × 103 = –2 × 96500 × Ecell


°

Mass of Fe deposited in FeSO4


_______________________ E°cell = 1.591 V
Mass of Fe deposited in Fe2(So4)3 58.7
____
144. The equivalent wt. of Ni = 2 = 29.35
M/2 3
= ____ __
M/3 = 2 The no. of equivalents of Ni deposited
Mass of Fe deposited in Fe(NO3)3 9.9
_____
________________________ = 29.35 ≈ 0.336 ≡ 0.335 F
Mass of Fe deposited in FeSO4
M/3 No. of equivalents of H2 liberated
= ____ 2
__
M/2 = 3
2.51
____
= 22.4 × 2 = 0.0224 ≡ 0.224 F
Mass of Fe deposited in Fe2(SO3)3
_________________________ Current efficiency for the deposition of Ni
Mass of Fe deposited in FeCl3 0.336
__________ 0.336
_____
= 0.336 + 0.224 = 0.56 = 0.6
M/3
____
= M/3 = 1 = 60 %

More than One Option Correct Type

148. Eocell with negative value means the non­spontaneous nature 153. (I) 2KI + Cl2 → 2KCl + I2
of the reaction in the specified direction. (II) 2KClO3 + I2 → 2KIO3 + Cl2
150. MnO4– + 8H+ + 5e– → Mn2+ + 4H2O; (III) 2KNO3(s) + 10K(s) → 6K2O (s) + N2
Eo = + 1.52 V (IV) 2P + 3I2 + 6 H2O → 2H3PO3 +6HI
Cr2O72– + 14H+ + 6e– → 2Cr3+ + 7H2O; Any reaction having one of the substance in its ele­
Eo = + 1.33 V mental form is a redox reaction. In disproportionation
The standard reduction potential of KMnO4 is higher than reaction same atom of the substance undergoes oxida­
that of K2Cr2O7. Hence KMnO4 is a stronger oxidising agent tion as well as reduction.
than K2Cr2O7 In a displacement reaction, an atom in a compound is
151. 3Cl2 + 6NaOH → 5NaCl + NaClO3 + 3H2O replaced by an atom of another element.
(hot)
Cl2 → Cl–1 [O.N. from 0 to −1]
Cl2 → ClO3− [O.N. from 0 to +5]

Passage Based Questions


Passage-1 0.059 [H 2Q]
E = Eo − 2log
155. Q + 2H + 2e +
H2Q 2 [Q][H + ]2
The Nernst equation for the electrode is Since [H2Q] = [Q]
The equation becomes
Redox Reactions and Electrochemistry 9.45

0.059 Passage-2
E = Eo + × 2log [H + ]
2 158. L Pb(s) + 2Cl− PbCl2(s) + 2e−
Or E = Eo + 0.059 log [H+] R 2AgCl(s) + 2e− 2Ag(s) + 2Cl−
156. Metallic ions Eored Eoox __________________________________
X 0.50 V − 0.50 V Pb(s) + 2AgCl(s) 2Ag(s) + PbCl2(s)
Y −3.03 V +3.03 V ΔG = − nEF = − 2(96500) (0.490 V)
Z −1.2 V + 1.2 V = −94570 J = − 94.6 kJ mol­1
Reducing power of the corresponding metal α Eoox  ∂E 
Hence reducing power of the metals follows the order Y > Z 159. ΔS = ∆S = nF  
∂T P
>X
E = a – (1.86 × 10–4 VK–1) (T – 25K)
157. According to the Faraday’s second law of electrolysis,
Mass of metal deposited α Chemical equivalent weight ∂E
= − 1.86 × 10–4 VK–1
∂T
At.Weight
For FeCl2, Equivalent weight of Fe = ΔS = 2(96500 C mol–1) (–1.86 × 10­4 VK–1)
2
M = − 35.9 J K–1 mol–1
=
2 160. ΔG = ΔH − TΔS
M ΔH = ΔG + TΔS
m2 α =
2
298
At. Weight = − 94.6 + (−35.9)
For FeCl3, equivalent weight = 1000
3
= − 94.6 – 10.67 kJ mol–1
M
= − 105.27 kJ mol–1
3
M Passage-3
m2 α 3 162. The feasible reaction is
m1 M 3 3 Cu(s) + 2Ag+ (aq) Cu+ (aq) + 2Ag (s) as
= × = E = 0.8 – 0.34 = +ve
o
m2 2 M 2 cell

or m1 : m2 = 3 : 2

Match the Following


165. [electro] 2H+ + 2e H2
Charge on one mole of electron 22.4 L of H2 at STP = 1 mole
= 16 × 10­19 × 6.023 × 1023 = 2 mole of electrons = 2 F
= 96500 Cb = 1 F 8 gram of O2 = one gram equivalent = 1F
Ag+ + e → Ag = 96500 Cb
1 mole of Ag required one mole of electrons 8 gm of O2 = 5.6 L of oxygen at STP
= 96500 Cb = 1 F

Integer Type
167. Cr2O72 + 14H+ + 6e− → 2Cr3+ + 7H2O
= 1.33 + 0.059 log 20–28 log 10
0.059 [Cr O ][H ] 2− + 14 6
E = E0Cr6+/Cr3+ + log 2 7 3+ 2
6 [Cr ] 0.059
= 1.33 + (1.3010–28)
6
(4.5 / 1000)(10 −2 )14 = 1.07 V ≈ 1.0 V
= 1.33 + 0.059 log
6 (15 / 1000) 2 168 Pdn+ + ne− → Pd
0.059 W i×t
= 1.33 + log 20 ×10–28 For Pd, =
6 E 96,500
9.46 Chapter 9

2.977 6 × 0.5 × 60 × 60 0.5 ln 0.5


= = log
106.4 96,500 0.059 2.575 × 10-18
n
= − 0.3010 – 0.4108 + 18 = 17.288
n = 4
17.288 × 0.059
169 96500 coulombs will deposit 108 g of Ag. n = = 2.0
0.51
8 × 96500
Therefore 8.0 g of Ag will be deposited by = MV 0.1 × 40
coulombs 108 172. Number of moles of hypo = =
1000 1000
Since time required is 1 hr
96500 × 8 1 = 4 × 10–3
Current in ampere required = ×
108 1 × 60 × 60 Number of moles of I2 = 2 × 10–3
Mass of I2 = 2 × 10–3 × 254 g
965 × 8 107 8
= ≈ × ≈ 2 amperes w =
i×t×e
; 2 × 10–3 × 254 =
i × 9650 × 127
108 × 36 108 4 96500 96500
170. Cu2+ + 2e Cu
i = 0.04 ampere.
0.059 1 170 Ag+(aq) + Fe2+ (aq) Ag(s) + Fe3+(aq)
E = E – 0
log 2+ or
2 Cu No. of 500 ×0.15 500×1.09 0 0
When E = 0 and E0 = 0.34 V mmoles = 75 = 545
0 = 0.34 – 0.059 log 1 Initially
2 [Cu 2+ ] mmoles at 75 – x 545 – x x x
Equilib.
0.059 1
Or log = 0.34 On titration of reaction mixture with KMnO4, only Fe2+
2 [Cu 2+ ] reacts with it.
Equivalents of Fe2+ in 25 ml = equivalents of KMnO4
1 0.34 × 2 0.68 × 10
log = = (545 − x)
Cu 2+ 0.059 0.59 × 25 × 10-3 × 1
1000
= 1.1525 × 10 = 11.525
−log[Cu2+] = 11.525 30 × 10–3 × 0.0833 × 5

Log [Cu2+] = 12.4756 (545 − x) 25


≈ 12.5
1000
Or [Cu2+] ≈ 3.0 × 10–12 M [log 3 = 0.4771]
545 – x = 500; x = 45
171. S + Bn+ Sn+ + B
Moles at t = 0 0.5 0 45
[Fe3+ ] 1000
Moles at equil. 2.575 ×10–18 0.5 –2.575×10­18 ∴ Kc =
[Ag + ][Fe 2+ ] (75 − 45) (545 − 45)
≈ 0.5
1000 1000
Ecell = EBn+/B– ESn+/S
45 × 1000
= =3
0.059 [Sn+ ] 30 × 500
= E Bn+/B– E
0 0
n+
S /S − log10 n+1
n [B ]
But, at equilibrium, Ecell = 0
0.059 0.5
0 = 0.24 – (−0.75) – log
n 2.575 × 10-18
Redox Reactions and Electrochemistry 9.47

Previous Years’ Questions


174. LHS half cell Given
H2 (g) 2H+ (1M) + 2e– E°(Sn2+/Sn) = – 0.14 V
P1 E°(Sn/Sn2+) = + 0.14 V
RHS half cell
E°(Fe3+/Fe2+) = 0.77 V
2H+ (1M) + 2e– H2 (g)
E°cell = 0.14 + 0.77 = 0.91 V
P2
H2 (g) H2 (g) 185. Relation between Keq and Ecell is
P1 P2 2.303 RT
_______
E°cell = nF log Keq
p__
2
E°cell = 0.00 V, K = p , n = 2 0.0591
_____
E°cell = n log Keq
1
RT
___ p__
2
Ecell = E°cell – nF loge K = 0 – RT loge p 0.0591
_____
1
0.591 = 1 log Keq
RT
___ p
__
Ecell = 2F log e p1 log Keq = 10
2
175. Oxidation takes place at anode.
Keq = 1 × 1010
Cr3+ is not oxidized to Cr2O2–
7
under given condition.
177. ZnO + C Zn + CO, ΔGO = –ve 186. By Kohlrausch’s law
So this is spontaneous. ΛNaBr
°
= Λ°NaCl + Λ°KBr – ΛKCl
°

178. S ∝ area (m2) = 126 + 152 – 150


∝ Concentration (mol/ m ) 3
= 128 S cm2 mol–1
1
_____
∝ (m–1) 187. Zn(s) + 2H+ Zn2+ (aq) + H2 (g)
length
mol
___
S = k m2 m3 m–1  Zn 2 + 
k = S mol–1 m2 Reaction quotient Q =  2
 H + 
0.0591
_____
179. Ecell = E°cell – n log Q
corresponding cell is
Cu + Zn Zn + Cu 0.0591
_____
E°cell = E°cell – 2
log K
0.1 M 1M [Zn2+]
0.0591
_____ _____
[Zn2+] ___
_____ 1 = E°cell – 2 log [H+]2
Q = [Cu2+] = 0.1 = 10
0.0591
_____ 0.0591
_____ [H+]
_____
2

Ecell = 1.10 – 2 log 10 = E°cell + 2


log [Zn2+]2
= 1.10 – 0.0295 If H2SO4 is added to cathodic compartment, then Q decreases
= 1.0705 V (due to increase in H+). So, equilibrium is displaced towards
2.303 RT
_______ right and Ecell increases.
180. E°cell = nF log Keq
0.0591
_____ 188. E°(Cr3+/Cr2+) = –0.41 V
0.295 = 2 log Keq
E°(Mn3+/Mn2+) = + 1.57 V
log Keq = 10
E°(Fe3+/Fe2+) = + 0.77 V
Keq = 1010
E°(Co3+/Co2+) = + 1.97 V
181. Ag+ + e– Ag
More negative value of E°red indicates better reducing agent,
9650 C = 0.1 F = 0.1 equivalent Ag this easily oxidized. So, oxidation of Cr2+ to Cr3+ is the
= 0.1 mol Ag = 10.8 g of Ag easiest.
189. For spontaneous process,
182. More negative value of E° implies greater reducing power.
ΔG = –ve, K > 1 and E°cell = +ve
183. Any cell (like fuel cell), works when potential difference is
m FZ
____
developed. 190. Q = M
184. Sn(s) + 2Fe3+ (aq) 2Fe2+ (aq) + Sn2+ (aq) 5120 × 96500 × 3
_____________
= 27
= 5.49 ×107 C of electricity
E°cell = E°ox + E°red 191. Λ∝(AcOH) = Λ∝ (AcONa) + Λ∝ (HCl) – Λ∝ (NaCl)
= E° (Sn/Sn ) + E° (Fe / Fe )
2+ 3+ 2+
= 91.0 + 426.2 – 126.5
= 390.7 S cm2 mol–1.
9.48 Chapter 9

192. Difluoroacetic acid will be strongest acid, due to electron ΔG0f H2O (l) = –237 kJ mol–1
withdrawing effect of two fluorine atoms, so it will show ΔG0f CO2 (l) = –394.4 kJ mol–1
maximum electrical conductivity. ΔG = ΣΔG0f products – ΣΔG0f reactants.
0.0591
_____
193. E° (AgI/Ag) = E (Ag+ /Ag) + 1 log Ksp AgI = –394.4 – 2 (237.2) + 166.2
0.0591
_____ = –702.6 kJ mol–1
–0.152 = + 0.8 + 1 log Ksp AgI
log Ksp (AgI) = –16.13 now efficiency of fuel cell

194. C = 0.1 M, R = 100 ohm, K= 1.29 ohm–1 m–1 ∆G


= × 100
l/a = K. R = 1.29 × 100 = 129 m –1 ∆H
702.6
C = 0.02 M, R = 520 = × 100
1
__ _l 1
___ 726
K = R × a = 320 × 129 = 0.248 ohm–1 m–1
= 97%
K decreases with dilution,
C = 0.02 and not 0.2 199. The value of E0 2+ for given metal ions are
M /M
1
________ 1
____________
Λ = K × M(in m/L) Λ = K × M × 103 (m/m3)
E0M 2+ / M = −1.18V , E0Cr 2+ / Cr = −0.9V ,
0.248
________
= 0.02 × 103 = 12.4 × 10–3 Sm2 mol–1
E0Fe2+ / Fe = −0.44 V and E0Co2+ / Co = −0.28V
= 124 × 10–4 S m2 mol–1

195. From Kohlrausch’s la The correct order of E0 2+ values without considering neg­
M /M

Λ° (CH3COOH) = Λ° (CH3COONa) + Λ° (HCl) – Λ° (NaCl) ative sign would be Mn2+ > Cr2+ > Fe2+ > Co2+.
0.0591
_____ 200. Here 2Al3+ 2Al – 6e–
196. Ecell = E°cell – n log10 Q
As ΔG = nF Ecell
[Zn2+]
_____
Here, Q = [Cu2+]
3
966 × × 103 = 6 × 96500 × Ecell
For complete discharge Ecell = 0. 2
0.0591
_____ [Zn2+]
_____ on solving Ecell = 2.5 V
So, E°cell = 2 log10 [Cu2+]
[Zn2+]
_____ 201. Pt | H2 (g) | H+ (aq)
[Cu2+]
= 1037.3
H+ + e– → 1/2 H2 (g)
197. As E°Cr3+ / Cr = –0.72 V and E°Fe2+ / Fe = –0.42V
( )
1/ 2
0.059 PH
E = E − log 2 + ;
2Cr + 3Fe2+ 3Fe + 2Cr3+ 1  H 
0.0591
_____ (Cr3+)2
_____
ECell = E°cell – 6
log10 (Fe2+)3 0.059 21/ 2
E=0− log = − ve
0.0591
_____ (0.1)
_____
2
1 1
= (–0.42 + 0.72) – 6 log10 (0.01)3
202. Order of elements in
0.0591
_____ (0.1)
_____
2

= 0.30 – 6 log10
(0.01)3
E.C.S. is Zn, Fe, Ni
0.0591
_____ 10­2
____ Zn can reduce Fe or Ni
= 0.30 – 6 log10 10­6
Zn(s) + Ni+2 → Zn+2 + Ni
= 0.30 – 0.0591/6 log10 104 X = Zn, Y = Ni
1 l
On solving, we get 204. 50 = ×
K A
Ecell = 0.2606 V. 1 l
50 = ×
198. CH3OH (l) + 3/2 O2 (g) CO2 (g) + 2H2O (l) 1.4 A
l 1
ΔH = –726 kJ mol–1 = 79 m −
A
Also
ΔG0f CH3OH (l) = –166.2 kJ mol–1
Redox Reactions and Electrochemistry 9.49

1 206. Mn +2 + 2e − → Mn E0 = −1.18V
280 = × 70
K
1 2Mn +2 → 2Mn +3 + 2e − E0 = −1.21V
K= Sm −1
4 3Mn +2 → Mn + 2Mn +3 E0 = SOP + SRP
1 1000  −2
Am = × (10 m) 2 = −1.18 + (−1.51)
4  M 
= −2.69 V
1 1000 −6
= × ×
4 0.5 Negative EMF reflects non­spontaneous cell reaction.
−6 −4 −1
= 500 × 10 = 5 × 10 2
Sm mol 207. Cu2+ + Ze → Cu
205. According to Debye Huckel’s theory for a strong electrolyte, 2 mole deposit 1 mole of Cu
2F ⇒ 2 mole → 1 mole of Cu ⇒ 63.5 gm.
λC = λ∞ − B C

You might also like